You are on page 1of 50

MULTIPLE CHOICE

1. Nurses with a bachelor’s degree in nursing can participate in the implementation of evidence-
based protocols in practice. This means that the BSN nurse
a. Develops evidence-based guidelines
b. Designs research studies, on which protocols may be based
c. Evaluates and revises evidence-based protocols
d. Contributes practice wisdom when applying protocols in patient settings
e. Mentors PhD researchers in the clinical setting during protocol development
ANS: D
Nurses with a BSN degree have knowledge of the research process and skills in reading and
critically appraising studies. They assist with the implementation of evidence-based
guidelines, protocols, algorithms, and policies in practice. This implies that nurses provide
their point of view, from the clinician’s vantage, when new protocols are being put into
practice, and continue to provide feedback, regarding the positive and negative aspects of
those protocols.

DIF: Cognitive Level: Analysis REF: Page 4

2. Research is designed to test the idea of providing companion dogs to elders in a major
hospital, in order to determine the effect upon the elders’ level of orientation. (The dogs’ level
of orientation will not be a focus of the research.) This type of study can do which of the
following?
a. Control
b. Describe
c. Explain
d. Predict
ANS: A
Control is the ability to manipulate the situation to produce the desired outcome. Description
involves identifying and understanding the nature of nursing phenomena and, sometimes, the
relationships among them. Explanation clarifies the relationships among phenomena and
identifies the reasons why certain events occur. The ability to estimate the probability of a
specific outcome in a given situation in nursing practice is known as prediction. The
researcher’s focus is on predicting what is likely.

DIF: Cognitive Level: Application REF: Page 13

3. A researcher wants to find out whether children with autism who are hospitalized on a
pediatric ward will require more hours of nursing care than average children when the parents
or caregivers are not present. What type of research outcome does this provide?
a. Control
b. Description
c. Explantation
d. Prediction
ANS: D
Control is the ability to manipulate the situation to produce the desired outcome. Description
involves identifying and understanding the nature of nursing phenomena and, sometimes, the
relationships among them. Explanation clarifies the relationships among phenomena and
identifies the reasons why certain events occur. The ability to estimate the probability of a
specific outcome in a given situation in nursing practice is known as prediction. The
researcher’s focus is on predicting what is likely.

DIF: Cognitive Level: Application REF: Page 16

4. A researcher who desires to determine the cause-and-effect relationship between requiring


that all children under the age of 8 will ride in special care harnesses and the subsequent rate
of children’s spinal cord injury will consequently utilize which form of nursing research?
a. Descriptive research
b. Outcomes research
c. Qualitative research
d. Quantitative research

ANS: D
Quantitative research, the most frequently used method, is a formal, objective, systematic
methodology to describe variables, test relationships, and examine cause-and-effect
interactions. Quantitative research includes experimental research, which is the method for
testing cause-and-effect relationships between and among specific variables. Qualitative
research methods are used for explaining meanings and describing experiences in context.
Descriptive research involves identifying and understanding the nature of phenomena and,
sometimes, the relationships among them. Outcomes research examines the end result of care
in huge populations, most often retrospectively, using a database.

DIF: Cognitive Level: Application REF: Page 3

5. Despite the presence of an intraventricular drain, the intracranial pressure of an ICU neuro
patient remains increased. The nurse recalibrates the machine, makes sure the monitor is on
the same level as the drain, checks all connections, and then notifies the physician, who comes
to the unit and inserts a new drain. What type of reasoning prompts the nurse to recalibrate,
ensure proper placement, and check connections?
a. Abstract reasoning
b. Concrete thinking
c. Logistic reasoning
d. Reality testing
ANS: C
Logistic reasoning is used to break a whole into parts that can be carefully examined.
Concrete thinking is oriented toward and limited by tangible things or by events that are
observed and experienced in reality. Abstract reasoning is oriented toward the development of
an idea without application to, or association with, a particular instance. Reality testing is used
to validate what is observed in the empirical world.

DIF: Cognitive Level: Application REF: Page 6


6. A nurse with considerable clinical expertise develops a policy for managing agitated patients
in the emergency department. The resultant policy emanates from
a. Abstract reasoning
b. Concrete thinking
c. Logistic reasoning
d. Reality testing
ANS: A
Abstract reasoning is oriented toward the development of an idea without application to, or
association with, a particular instance. Concrete thinking is oriented toward and limited by
tangible things or by events that are observed and experienced in reality. Logistic reasoning is
used to break a whole into parts that can be carefully examined. Reality testing is used to
validate what is observed in the empirical world.

DIF: Cognitive Level: Application REF: Page 7

7. A nurse with considerable clinical expertise develops a policy for managing agitated patients
in the emergency department. The type of reasoning the nurse uses to do this is _____
reasoning.
a. Problematic
b. Operational
c. Logistic
d. Inductive
ANS: D
Inductive reasoning involves reasoning that moves from the specific to the general, whereby
particular instances are observed and then combined into a larger whole or general statement.
Problematic reasoning involves (1) identifying a problem and factors influencing it, (2)
selecting solutions to the problem, and (3) resolving the problem. Operational reasoning
involves the identification of and discrimination among many alternatives and viewpoints.
Logistic reasoning is used to break the whole into parts that can be carefully examined, as the
relationships among the parts can also be.

DIF: Cognitive Level: Synthesis REF: Page 7

8. What is the best explanation of intuition that forms a legitimate source of knowledge in
nursing?
a. It is based on knowledge thoroughly incorporated into thought but seldom
articulated.
b. It is based on a gift from the universe and should be honored when it arrives.
c. It is never inaccurate.
d. It is a revisiting of old knowledge, accompanied by deep reflection.
ANS: A
Intuition is the revisiting of old knowledge accompanied by deep reflection.

DIF: Cognitive Level: Application REF: Page 5

9. Why is operational reasoning necessary for research?


a. Abstract concepts are of no use to nursing.
b. Standard interventions are obtained from operational reasoning.
c. It allows the researcher to measure the concepts studied.
d. It facilitates the researcher’s rapport with families.
ANS: C
Operational reasoning involves the identification of and discrimination among many
alternatives and viewpoints. It focuses on the process (debating alternatives) rather than on the
resolution. Nurses use operational reasoning to develop realistic, measurable health goals.
Thus, operational reasoning takes abstract concepts and makes them focused, concrete, and,
therefore, researchable.

DIF: Cognitive Level: Application REF: Page 6

MULTIPLE RESPONSE

1. What are the connections between evidence-based practice and nursing research? (Select all
that apply.)
a. Evidence-based care cannot be provided to patients without the nurse
understanding something of research.
b. A synthesis of current research within an area of nursing is used to improve care in
that area.
c. All patients with a given diagnosis should be cared for based solely on research
knowledge.
d. Nursing diagnosis and management depend on a practitioner’s exploration of best
research evidence.
e. Nursing research provides evidence that allows us each to practice with the same
style and capability.
ANS: A, B, D
Evidence-based practice in nursing requires a strong body of research knowledge that nurses
must synthesize and use to promote quality care for their patients, families, and communities.
In order to synthesize and use research appropriately, a nurse must understand it. A nurse
must explore the best research evidence about a practice problem before using his or her
clinical expertise to diagnose and manage an individual patient’s health problem. Not all
patients are treated in the same way, however. Because reality can vary with perception, and
because the facts can be relative, nurses do not impose their views on patients. Rather, nurses
help patients seek health from within the patients’ worldviews. This is a critical component of
evidence-based practice.

DIF: Cognitive Level: Analysis REF: Page 11

2. What is the hospitalized patient’s place in evidence-based practice? (Select all that apply.)
a. The patient is the recipient of the total of formal research evidence and the nurse’s
practice wisdom, and these represent the patient’s care plan.
b. The patient brings values to the clinical encounter, which the nurse considers in
providing evidence-based care.
c. The patient provides a valuable source of knowledge, since each patient cared for
contributes to the nurse’s total practice wisdom.
d. The patient is the focus of research. The patient serves both as a recipient of
evidence-based research and the subject of future evidence, based on data collected
now from the patient.
e. The patient may always refuse to participate—in evidence-based care, in therapies,
in research participation—and this refusal must be honored.
ANS: B, C, E
Because reality can vary with perception, and because the facts can be relative, nurses do not
impose their views on patients. Rather, nurses help patients seek health from within the
patients’ worldviews. This is a critical component of evidence-based practice. The nurse’s
individual wisdom is based upon the nurse’s actual practice, over time.

DIF: Cognitive Level: Analysis REF: Page 11

3. In nursing mentorship, as opposed to authority, the novice nurse fills which of the following
roles? (Select all that apply.)
a. Counselor
b. Student
c. Sponsor
d. Disciplinarian
e. Teacher
f. Questioner
g. Apprentice

ANS: B, F, G
An intense form of role-modeling is mentorship. In a mentorship, the expert nurse—or
mentor—serves as a teacher, sponsor, guide, exemplar, and counselor for the novice nurse.
Over time, the relationship morphs into a colleague relationship in which both mentor and
mentee share information and exchange ideas in a cooperative spirit.

DIF: Cognitive Level: Analysis REF: Page 10

4. How are dialectic reasoning and holistic practice similar? (Select all that apply.)
a. They are both based on intuition, not facts.
b. They both consider the whole, rather than one part of the picture.
c. Dialectic reasoning emphasizes truth and holistic practice accepts untruth.
d. They both ignore the main idea or diagnosis and concentrate on different entities.
e. They both honor context and the interactions among ideas and people.
f. They both break down concepts into understandable parts.
g. Dialectic reasoning can be used to validate a study design, whereas holistic
practice does not contribute to research.
ANS: B, E, F
Dialectic reasoning involves looking at situations in a holistic way. A dialectic thinker
believes that the whole is greater than the sum of the parts and that the whole organizes the
parts.

DIF: Cognitive Level: Synthesis REF: Page 6

5. Which of the following statements synopsize the relationship between research and practice?
(Select all that apply.)
a. Practice limits nursing research to issues that are client-centered.
b. Research participation helps nurses to meet re-licensure requirements for evidence-
based practice.
c. Research knowledge, combined with experiential wisdom, constitutes the base for
practice.
d. Research emphasizes what can be done in practice, rather than what has been done
in practice.
e. Practice does not affect research: research affects practice.
f. Practice provides inspiration for meaningful nursing research.
g. Practice helps a nurse differentiate between rigorous, well-designed research and
useless research.
ANS: C, F
Evidence-based practice in nursing requires a strong body of research knowledge that nurses
must synthesize and use to promote quality care for their patients. Research is a way to test
reality and generate the best evidence to guide nursing practice. Practice problems inspire
meaningful clinical research. Evidence-based practice evolves from the integration of the best
research evidence with clinical expertise and patient needs and values.

DIF: Cognitive Level: Evaluation REF: Page 3

6. Realistically, what might be done in a situation in which a nurse does not know the
appropriate way to use a new ultrasonic bladder scanner (a non-invasive, painless procedure)
but has an order to scan? (Select all that apply.)
a. Refuse to carry out the order.
b. Ask a co-worker who has used the equipment for help.
c. Access the instructions on the company’s web site.
d. Try to scan the bladder and see if the value obtained makes sense.
e. Notify the manager that a formal inservice is needed.
f. Read the instruction booklet.
ANS: B, C, D, F
Trial and error is an approach with unknown outcomes that is used in a situation of
uncertainty, when other sources of knowledge are unavailable. The profession evolved
through a great deal of trial and error before knowledge of effective practices was codified in
textbooks and journals. The trial-and-error way of acquiring knowledge can be time-
consuming, because multiple interventions might be implemented before one is found to be
effective.

DIF: Cognitive Level: Application REF: Page 9

7. Which of the following sources generates new knowledge for nurses? (Select all that apply.)
a. Editorials in nursing journals
b. Qualitative research
c. Adhering to hospital policies
d. Research that tests a new sling scale for safety of patients and nurses
e. Quantitative research
f. Comparison of two different insulin dosing protocols
ANS: B, D, E
Nursing research is defined as a scientific process that validates and refines existing
knowledge and generates new knowledge that directly and indirectly influences the delivery
of evidence-based nursing. Nurses use a variety of research methods to test their reality and
generate nursing knowledge, including quantitative research, qualitative research, outcomes
research, and intervention research.

DIF: Cognitive Level: Application REF: Page 2

8. Which of these are suitable focuses for a nursing research study? (Select all that apply.)
a. How clinical nurse specialists contribute to patient outcomes
b. Which elements of a nursing school curriculum remain useful for current practice
after students graduate
c. Whether requiring nurse managers to supervise more than four units is cost-
effective
d. What styles of physician teaching produce better diabetic compliance
e. Whether patients with exacerbation of CHF are best managed with inpatient
treatment or with outpatient treatment
f. What the personality characteristics are of nurses in various inpatient areas
ANS: A, B, C, F
Many nurses hold the view that nursing research should focus on acquiring knowledge that
can be directly implemented in clinical practice, which is sometimes referred to as applied
research or practical research. However, another view is that nursing research should include
studies of nursing education, nursing administration, health services, and nurses’
characteristics and roles, as well as clinical situations, education, practice, and service.
Research is needed to identify teaching-learning strategies to promote nurses’ management of
practice. Thus, nurse researchers are involved in building a science for nursing education so
that the teaching-learning strategies used are evidence-based. Nurse administrators are
involved in research to enhance nursing leadership and the delivery of quality, cost-effective
patient care.

DIF: Cognitive Level: Application REF: Page 1

9. What might a nursing research study address? (Select all that apply.)
a. Whether having a nurse practitioner manage care is effective in decreasing length-
of-stay
b. Whether students learn better in an online course format or by actual lecture
attendance
c. Comparing four types of leadership used by nurse managers and comparing their
employees’ job satisfaction, absenteeism rates, and error rates
d. Different common surgical procedures and the mortality rate of each
e. Learning specific things about the liver failure patient that can be applied to
nursing practice
ANS: A, B, C, E
Many nurses hold the view that nursing research should focus on acquiring knowledge that
can be directly implemented in clinical practice, which is sometimes referred to as applied
research or practical research. However, another view is that nursing research should include
studies of nursing education, nursing administration, health services, and nurses’
characteristics and roles, as well as clinical situations, education, practice, and service.
Research is needed to identify teaching-learning strategies to promote nurses’ management of
practice. Thus, nurse researchers are involved in building a science for nursing education so
that the teaching-learning strategies used are evidence-based. Nurse administrators are
involved in research to enhance nursing leadership and the delivery of quality, cost-effective
patient care.

DIF: Cognitive Level: Application REF: Page 1

MULTIPLE CHOICE

1. In which way did Florence Nightingale contribute to evidence-based practice?


a. She conducted research on outcomes and the power of nursing for change.
b. She was the first woman elected to the Royal Statistical Society.
c. She gathered data that changed the care of hospitalized soldiers.
d. She calculated mortality rates under varying conditions.
ANS: C
Nightingale gathered data on soldier morbidity and mortality rates and the factors influencing
them and presented her results in tables and pie charts, a sophisticated type of data
presentation for the period. Nightingale’s research enabled her to instigate attitudinal,
organizational, and social changes. She changed the attitudes of the military and society
toward the care of the sick. The military began to view the sick as having the right to adequate
food, suitable quarters, and appropriate medical treatment, which greatly reduced the mortality
rate.

DIF: Cognitive Level: Application REF: Page 17

2. If a nurse manager wants to study how well last year’s policies governing implementation of a
“bundle” of interventions to prevent cross-contamination of MRSA have been working in her
units, which of the following strategies would she use?
a. Outcomes research
b. Intervention research
c. Ethnographic research
d. Experimental research
ANS: A
Outcomes research emerged as an important methodology for documenting the effectiveness
of health care services in the 1980s and 1990s. This type of research evolved from the quality
assessment and quality assurance functions that originated with the professional standards
review organizations (PSROs) in 1972. During the 1980s, William Roper, the director of the
Health Care Finance Administration (HCFA), promoted outcomes research for determining
the quality and cost-effectiveness of patient care. Intervention research investigates the
effectiveness of a nursing intervention in achieving the desired outcome or outcomes in a
natural setting. Through the use of ethnographic research, different cultures are described,
compared, and contrasted to add to our understanding of the impact of culture on human
behavior and health. Experimental studies have three main characteristics: (1) a controlled
manipulation of at least one treatment variable (independent variable), (2) administration of
the treatment to some of the subjects in the study (experimental group) and not to others
(control group), and (3) random selection of subjects or random assignment of subjects to
groups, or both. Experimental studies usually are conducted in highly controlled settings, such
as laboratories or research units in clinical agencies.

DIF: Cognitive Level: Analysis REF: Page 22

3. A researcher publishes a paper describing how faith, pain, adherence to therapy, and
meditation interact during the rehabilitation process. The description of the process is based
on many interviews the researcher conducted with persons during and following rehabilitation
experiences. The methodology is
a. Ethnography
b. Phenomenology
c. Historical research
d. Grounded theory
ANS: D
Grounded theory methodology emphasizes observation and the development of practice-based
intuitive relationships among variables. Throughout the study, the researcher formulates, tests,
and redevelops propositions until a theory evolves. The theory developed is “grounded,” or
has its roots in, the data from which it was derived.

DIF: Cognitive Level: Synthesis REF: Page 27

4. A panel of researchers conducts several studies, all drawn from an existent hospital and clinic
database. The studies focus on quality and effectiveness within that system. The specific
studies address mortality rates in elders within a year after hip fracture, functional outcomes
six months after admission to a neurosurgical ICU after traumatic brain injury, rate of nurse
injuries in an emergency department, and number of patient falls on various floors of the
hospital. What type of research is this?
a. Experimental research
b. Outcomes research
c. Ethnographic research
d. Grounded theory research
ANS: B
The spiraling cost of health care has generated many questions about the quality and
effectiveness of health care services and the patient outcomes. Consumers want to know what
services they are buying, and whether these services will improve their health. Health care
policy makers want to know whether the care is cost-effective and of high quality. These
concerns have promoted the development of outcomes research, which examines the results of
care and measures the changes in health status of patients. It can also examine costs related to
care delivery within a hospital system.

DIF: Cognitive Level: Comprehension REF: Page 27

5. A researcher designs a study. It depends on questionnaires for data, it has a clear purpose
statement, it provides its results as a narrative without statistical analysis, and it makes general
suggestions for practice. What type of research is this?
a. Qualitative research
b. Outcomes research
c. Intervention research
d. Quantitative research
ANS: D
Quantitative research is a formal, objective, systematic process in which numerical data are
used to obtain information about the world. Qualitative research is also systematic, but it is a
holistic, interactive, and subjective approach to describe life experiences and identify their
meaning. Both types of research have a purpose statement and can use a survey instrument;
however, neither depends on surveys for data. Both can contain suggestions for practice.
Qualitative research results are presented as a narrative, without statistical analysis. Outcomes
research examines the results of care and measures the changes in health status of patients.
Intervention research investigates the effectiveness of a nursing intervention in achieving the
desired outcome or outcomes in a natural setting.

DIF: Cognitive Level: Application REF: Page 23

6. A newly employed nurse administrator wants to know more about the employees on the units
the administrator supervises. The manager accesses the managerial database and gathers data
about all of the current employees on the unit, including work shift, number of years
employed, age, gender, educational preparation, certifications, work history, and professional
accomplishments. What type of research is this?
a. Descriptive research
b. Correlational research
c. Quasi-experimental research
d. Experimental research
ANS: A
The quantitative research methods are classified into four categories: (1) descriptive, which
defines the magnitude of a concept and its characteristics, (2) correlational, which determines
association between or among variables, (3) quasi-experimental, which tests an intervention
and lacks control in at least one of three areas, and (4) experimental, which tests an
intervention and includes both a control group and random assignment. This is a research
study, even though it depends upon existent data, collected by another manager. Its purpose is
to describe the employees.

DIF: Cognitive Level: Analysis REF: Page 26


7. A human resources employee performs research focusing on the professional lifespan within
the institution of nurses, and trying to discover whether their choice of work area is connected
with the number of years they work in the institution. What type of research is this?
a. Descriptive research
b. Correlational research
c. Quasi-experimental research
d. Experimental research

ANS: B
The quantitative research methods are classified into four categories: (1) descriptive, which
defines the magnitude of a concept and its characteristics, (2) correlational, which determines
association between or among variables, (3) quasi-experimental, which tests an intervention
and lacks control in at least one of three areas, and (4) experimental, which tests an
intervention and includes both a control group and random assignment. This study
investigates the connection or association between work area and length of time worked.

DIF: Cognitive Level: Analysis REF: Page 26

8. In an attempt to assess whether selection of a same-gender psychiatrist leads to better mental


health outcomes, clients newly referred for mental health services are told they may choose
their mental health physicians. Later, measures of mental health are performed. What type of
research is this?
a. Descriptive research
b. Correlational research
c. Quasi-experimental research
d. Experimental research
ANS: C
The quantitative research methods are classified into four categories: (1) descriptive, which
defines the magnitude of a concept and its characteristics, (2) correlational, which determines
association between or among variables, (3) quasi-experimental, which tests an intervention
and lacks control in at least one of three areas, and (4) experimental, which tests an
intervention and includes both a control group and random assignment. This research study is
designed to test an intervention but does not include random assignment.

DIF: Cognitive Level: Analysis REF: Page 26

9. In a rehabilitation unit, patients are randomly assigned to high fiber diets versus ordinary fiber
diets, in order to measure the effect on constipation. What type of research is this?
a. Descriptive research
b. Correlational research
c. Quasi-experimental research
d. Experimental research
ANS: D
The quantitative research methods are classified into four categories: (1) descriptive, which
defines the magnitude of a concept and its characteristics, (2) correlational, which determines
association between or among variables, (3) quasi-experimental, which tests an intervention
and lacks either a control group or random assignment, and (4) experimental, which tests an
intervention and includes both a control group and random assignment. This research study
tests an intervention and includes both a control group and random assignment.

DIF: Cognitive Level: Analysis REF: Page 26

10. A researcher uses interviews with two or three open-ended questions to study women in the
staging phase of breast cancer treatment, in order to understand their experiences and the
meanings they attribute to those experiences. What type of research is this?
a. Phenomenologic research
b. Grounded theory research
c. Ethnographic research
d. Historicism
ANS: A
Phenomenologic research examines the lived experiences of participants and the meanings
those experiences hold for them, drawing its results only from the participants’ views.
Grounded theory research defines under-researched concepts and explains them within a
social framework, building on both observation and the perceptions of the persons who are
familiar with the concepts, and sometimes generating theory; it emphasizes interaction,
observation, and development of relationships among concepts. Ethnography defines shared
characteristics of members of a culture or participants who share in a common characteristic,
and explains commonalities, often within a cultural framework, using observation, interview,
and other data collection strategies; through the use of ethnographic research, different
cultures are described, compared, and contrasted to add to our understanding of the impact of
culture on the human experience. Historicism tells the story of past events, reconstructing
these from other historical references, interviews, artifacts, art, and other sources that reflect
the time of interest.

DIF: Cognitive Level: Analysis REF: Page 27

11. A researcher uses interviews with eight open-ended questions to study women in a new
staging phase of breast cancer treatment, which includes serial biopsies and necessitates
weekly closed biopsy, in order to understand more about social factors that impinge upon their
experience. What type of research is this?
a. Phenomenologic research
b. Grounded theory research
c. Ethnographic research
d. Historicism
ANS: B
Grounded theory research defines under-researched concepts and explains them within a
social framework, building on both observation and the perceptions of the persons who are
familiar with the concepts, and sometimes generating theory; it emphasizes interaction,
observation, and development of relationships among concepts. Phenomenologic research
examines the lived experiences of participants and the meanings those experiences hold for
them, drawing its results only from the participants’ views. Ethnography defines shared
characteristics of members of a culture or participants who share in a common characteristic,
and explains commonalities, often within a cultural framework, using observation, interview,
and other data collection strategies; through the use of ethnographic research, different
cultures are described, compared, and contrasted to add to our understanding of the impact of
culture on the human experience. Historicism tells the story of past events, reconstructing
these from other historical references, interviews, artifacts, art, and other sources that reflect
the time of interest.

DIF: Cognitive Level: Analysis REF: Page 27

12. A researcher conducts many interviews, over a one-year period, with women in the treatment
phase of breast cancer, all of whom are attending a breast cancer support group, in order to
understand what happens in the support group, how the members are affected by membership,
and how the members contribute to the group. The researcher herself is also in treatment for
breast cancer and is a member of the group. What type of research is this?
a. Phenomenologic research
b. Grounded theory research
c. Ethnographic research
d. Historicism

ANS: C
Ethnography defines shared characteristics of members of a culture or participants who share
in a common characteristic, and explains commonalities, often within a cultural framework,
using observation, interview, and other data collection strategies; through the use of
ethnographic research, different cultures are described, compared, and contrasted to add to our
understanding of the impact of culture on the human experience. Phenomenologic research
examines the lived experiences of participants and the meanings those experiences hold for
them, drawing its results only from the participants’ views. Grounded theory research defines
under-researched concepts and explains them within a social framework, building on both
observation and the perceptions of the persons who are familiar with the concepts, and
sometimes generating theory; it emphasizes interaction, observation, and development of
relationships among concepts. Historicism tells the story of past events, reconstructing these
from other historical references, interviews, artifacts, art, and other sources that reflect the
time of interest.

DIF: Cognitive Level: Analysis REF: Page 27

13. A researcher reviews the twenty years that a breast cancer clinic has been in operation in a
small Midwestern city. The researcher interviews many of the women who have been treated
in the clinic during this period and reviews the records of the clinic, along with its survival
rates and the emergence of several of its innovative support programs for women and their
families. The researcher ultimately writes a story of the clinic over those twenty years. What
type of research is this?
a. Phenomenologic research
b. Grounded theory research
c. Ethnographic research
d. Historicism
ANS: D
Phenomenologic research examines the lived experiences of participants and the meanings
those experiences hold for them, drawing its results only from the participants’ views.
Grounded theory research defines under-researched concepts and explains them within a
social framework, building on both observation and the perceptions of the persons who are
familiar with the concepts, and sometimes generating theory; it emphasizes interaction,
observation, and development of relationships among concepts. Ethnography defines shared
characteristics of members of a culture or participants who share in a common characteristic,
and explains commonalities, often within a cultural framework, using observation, interview,
and other data collection strategies; through the use of ethnographic research, different
cultures are described, compared, and contrasted to add to our understanding of the impact of
culture on the human experience. Historicism tells the story of past events, reconstructing
these from other historical references, interviews, artifacts, art, and other sources that reflect
the time of interest.

DIF: Cognitive Level: Analysis REF: Page 27

MULTIPLE RESPONSE

1. Florence Nightingale researched mortality and morbidity rates in soldiers during the Crimean
War and investigated various factors that influenced both, presenting her results as pie charts
and graphs. Consequently, it is known that she conducted which types of research? (Select all
that apply.)
a. Phenomenologic research
b. Causational research
c. Descriptive research
d. Correlational research
e. Ethnographic research

ANS: C, D
Nightingale is noted for her data collection and statistical analyses during the Crimean War.
She gathered data on soldier morbidity and mortality rates and the factors influencing them
and presented her results in tables and pie charts, a sophisticated type of data presentation for
the period. There is no evidence that she designed causational (experimental or quasi-
experimental) research or any type of qualitative research.

DIF: Cognitive Level: Analysis REF: Page 17

2. Which of the following statements about quantitative research is accurate? (Select all that
apply.)
a. The results of quantitative research should be generalized back to the population
from which the sample was drawn.
b. Quantitative research is always easy and straightforward to read and understand.
c. Quantitative research addresses quantities, connections, and causes.
d. Quantitative research predominates in the nursing research literature.
e. Quantitative research is always experimental.
f. Quantitative research provides answers to “What?” and “Who?” questions.
ANS: A, C, D, F
The quantitative approach to scientific inquiry emerged from a branch of philosophy called
logical positivism, which operates on strict rules of logic, truth, laws, axioms, and predictions.
Quantitative research requires the use of structured interviews, questionnaires, or
observations, scales, or physiological measures that generate numerical data. Statistical
analyses are conducted to reduce and organize data, describe variables, examine relationships,
and determine differences among groups. Control, instruments, and statistical analyses are
used to ensure that the research findings accurately reflect reality so that the study findings
can be generalized. Generalization involves the application of trends or general tendencies
(which are identified by studying a sample) to the population from which the research sample
was drawn. Researchers must be cautious in making generalizations, because a sound
generalization requires the support of many studies with a variety of samples.

DIF: Cognitive Level: Analysis REF: Page 24

3. Which of the following statements about qualitative research is accurate? (Select all that
apply.)
a. Qualitative research deals exclusively with humans.
b. Qualitative research’s principal purpose is to inform the reader.
c. Qualitative research yields data that are not numbers-based, such as audiotapes,
videotapes, and field notes.
d. Qualitative research is not systematic.
e. Qualitative research does not contain or imply a research question.
f. Qualitative research is ill-defined and vague.
g. Qualitative research has no practical use.
ANS: B, C
Qualitative researchers use observations, interviews, and focus groups to gather data. The
interactions are guided but not controlled in the way that quantitative data collection is
controlled. For example, the researcher may ask subjects to share their experiences of
powerlessness in the health care system. Qualitative researchers would begin interpreting the
subjective data during data collection, recognizing that their interpretation is influenced by
their own perceptions and beliefs. Qualitative data take the form of words and are analyzed
according to the qualitative approach that is being used. The intent of the analysis is to
organize the data into a meaningful, individualized interpretation, framework, or theory that
describes the phenomenon studied. The findings from a qualitative study are unique to that
study, and it is not the researcher’s intent to generalize the findings to a larger population.
Qualitative researchers are encouraged to question generalizations and to interpret meaning
based on individual study participants’ perceptions and realities.

DIF: Cognitive Level: Analysis REF: Page 25

4. Which is true of quantitative research? (Select all that apply.)


a. It addresses human responses by measuring or counting them.
b. It presents information by clustering it or counting it.
c. It yields a data set that can be analyzed by statistics.
d. It operates systematically.
e. It states or implies a research question.
f. It operates in a concrete realm.
g. It can always be generalized.
ANS: A, B, C, D, E, F
The quantitative approach to scientific inquiry emerged from a branch of philosophy called
logical positivism, which operates on strict rules of logic, truth, laws, axioms, and predictions.
Quantitative research requires the use of structured interviews, questionnaires, or
observations, scales, or physiological measures that generate numerical data. Statistical
analyses are conducted to reduce and organize data, describe variables, examine relationships,
and determine differences among groups. Control, instruments, and statistical analyses are
used to ensure that the research findings accurately reflect reality so that the study findings
can be generalized. Generalization involves the application of trends or general tendencies
(which are identified by studying a sample) to the population from which the research sample
was drawn. Researchers must be cautious in making generalizations, because a sound
generalization requires the support of many studies with a variety of samples.

DIF: Cognitive Level: Analysis REF: Page 24

5. Ethnographic research might focus upon which of the following topics? (Select all that apply.)
a. Bacterial cultures
b. Cultural beliefs of the ancient Romans
c. How children in Alaska play during the winter
d. Twenty-year abstinence members of Alcoholics Anonymous
e. The mentoring process in a labor-delivery unit
f. Conversational Spanish

ANS: C, D, E
Ethnographic research was developed by anthropologists to investigate cultures through an in-
depth study of the members of the culture. The culture may be an actual culture, a loosely
connected group of people who share a common characteristic, or a work or recreational
group. The ethnographic research process is the systematic collection, description, and
analysis of data to develop a description of cultural behavior. The researcher (ethnographer)
actually lives in or becomes a part of the cultural setting to gather the data.

DIF: Cognitive Level: Analysis REF: Page 24

6. A researcher is operating from the point of view of logical positivism. Which of the following
research methods would the logical positivist use? (Select all that apply.)
a. Grounded theory research
b. Correlational research
c. Historical research
d. Quasi-experimental research
e. Quantitative descriptive research
f. Exploratory descriptive qualitative research
ANS: B, D, E
The quantitative approach to scientific inquiry emerged from a branch of philosophy called
logical positivism, which operates on strict rules of logic, truth, laws, axioms, and predictions.
The quantitative research methods are classified into four categories: (1) descriptive, (2)
correlational, (3) quasi-experimental, and (4) experimental. The qualitative research methods
included in this textbook are (1) phenomenological research, (2) grounded theory research, (3)
ethnographic research, (4) exploratory-descriptive qualitative research, and (5) historical
research.

DIF: Cognitive Level: Analysis REF: Page 27

7. Which of the follow potential studies would fall within the Agency for Healthcare Research
and Quality’s future research goals? (Select all that apply.)
a. Performing a synthesis of research evidence regarding skin-to-skin contact of
mothers and newborns
b. Enacting a quantitative research project measuring bacterial count on nurses’
uniforms at the beginning and the end of 12-hour work shifts
c. Performing a qualitative research project to explain sources of student nurses’
stress
d. Enacting a public education Internet commercial encouraging smokers to read the
statistics regarding sequelae of cigarette smoking
e. Trialing clean-and-sober support groups that are based in community shopping
centers
ANS: A, D, E
The Agency for Healthcare Research and Quality partners with public and private sectors to
improve the quality and safety of patient care by promoting the use of the best research
evidence available in practice. Its three future goals are focused on the following: “Safety and
quality: Reduce the risk of harm by promoting delivery of the best possible health care;
Effectiveness: Improve healthcare outcomes by encouraging the use of evidence to make
informed healthcare decisions; and Efficiency: Transform research into practice to facilitate
wider access to effective healthcare services and reduce unnecessary costs.”

DIF: Cognitive Level: Analysis REF: Page 22

8. Early nursing research by Nightingale focused on improving patient outcomes. What were the
principal topics for the next wave of nursing research, in the first half of the 20th century?
(Select all that apply.)
a. Evidence-based practice
b. Primary nursing’s advantages in hospitals
c. Nursing education, as opposed to nurse training
d. The nursing process and nursing diagnosis
e. Staffing, patient assignments, and type of care
ANS: C, E
From 1900 to 1950, research activities in nursing were limited, but a few studies advanced
nursing education. Based on recommendations of the Goldmark Report, more schools of
nursing were established in university settings. A research trend that started in the 1940s and
continued in the 1950s focused on the organization and delivery of nursing services. Studies
were conducted on the numbers and kinds of nursing personnel, staffing patterns, patient
classification systems, patient and nurse satisfaction, and unit arrangement. Types of care such
as comprehensive care, home care, and progressive patient care were evaluated. In the 1970s,
the nursing process became the focus of many studies, with the investigations of assessment
techniques, nursing diagnoses classification, goal-setting methods, and specific nursing
interventions. Primary nursing care, which involves the delivery of patient care predominantly
by registered nurses (RNs), was the trend for the 1970s. The vision for nursing research in the
twenty-first century includes conducting quality studies using a variety of methodologies,
synthesizing the study findings into the best research evidence, and using this research
evidence to guide practice. The focus on EBP has become stronger over the last decade.

DIF: Cognitive Level: Synthesis REF: Page 19

9. Which of the following is true of the Cochrane Center and Cochrane Collaboration, begun in
the 1970s by Professor Archie Cochrane? (Select all that apply.)
a. It was originally called the Walter Reed Army Institute of Research.
b. It developed the original master’s degrees in nursing practice.
c. It serves as a repository for evidence-based practice guidelines.
d. It was the first association to publish a nursing research journal.
e. It is the online library resource for research literature reviews.
ANS: C, E
Cochrane advocated the provision of health care based on research to improve the quality of
care and patient outcomes. To facilitate the use of research evidence in practice, the Cochrane
Center was established in 1992 and the Cochrane Collaboration in 1993. The Cochrane
Collaboration and Library house numerous resources to promote EBP, such as systematic
reviews of research and evidence-based guidelines for practice.

DIF: Cognitive Level: Evaluation REF: Page 20

10. How does quantitative research contribute to evidence-based practice? (Select all that apply.)
a. It provides facts that nurses can add to their knowledge base. This makes practice
more objective and quantifiable.
b. It provides scientific support for policies already in place. More evidence makes an
existent policy more defensible.
c. It provides evidence opposing policies already in place. Evidence in opposition to
policies may result in new policies.
d. It allows the nurse to understand the personal experience of illness and the
meaning the client attaches to it. This engenders compassion.
e. It contributes evidence that will make nursing practice almost completely
evidence-based, eliminating different styles of nursing practice.
ANS: B, C
Quantitative research is a formal, objective, systematic process in which numerical data are
used to obtain information about the world. This research method is used to describe
variables, examine relationships among variables, and determine cause-and-effect interactions
between variables. The qualitative research method of phenomenology allows understanding
of the lived experience and the meaning it engenders. The aim of phenomenology is to
explore an experience as it is lived by the study participants and interpreted by the researcher.
Evidence-based practice is the conscientious integration of best research evidence with
clinical expertise and patient values and needs in the delivery of quality, cost-effective health
care. It provides the basis for policy decisions and for voluntary change in individual nursing
practice. Nursing style is a matter of personal choice.

DIF: Cognitive Level: Analysis REF: Page 23

11. What does appropriate generalization require? (Select all that apply.)
a. Any type of sample, whether or not it is representative
b. Application of findings to the population from which the sample was drawn
c. More than one research study using the same research questions and variables
d. Statistically significant findings
e. Non-significant findings that are supported by several related studies
ANS: B, C, D
Generalization involves the application of trends or general tendencies (which are identified
by studying a sample) to the population from which the research sample was drawn.
Researchers must be cautious in making generalizations, because a sound generalization
requires the support of many studies with a variety of samples.

DIF: Cognitive Level: Analysis REF: Page 23

12. What best characterizes the contribution of qualitative nursing research to evidence-based
practice? (Select all that apply.)
a. It presents collective common evidence of health care clients’ experiences, which
may provide inspirations for individual practice.
b. It provides stories of how health care clients feel. This lets nurses know what
people in similar circumstances can be expected to experience.
c. It provides evidence that determines how nurses should interact with various
cultures. This mandates action.
d. It generates and tests theory.
e. It reveals participants’ experiences and individual viewpoints, feelings, and
interpretations. These can provide guidelines for client-centered care.
ANS: A, E
Qualitative research is a systematic, interactive, subjective approach used to describe life
experiences from the research participants’ point of view. This type of research is conducted
to explore, describe, and promote understanding of human experiences, events, and cultures
over time. It is holistic and describes the human in context.

MULTIPLE CHOICE
1. A researcher conducts a study to determine the effectiveness of a special program of
sensitivity training for nurse managers upon several outcomes, all related to the staff’s ability
to identify and intervene appropriately when medication errors occur. This is an example of
what type of quantitative research?
a. Applied research
b. Basic research
c. Descriptive research
d. Qualitative research
ANS: A
Applied, or practical, research is a scientific investigation conducted to generate knowledge
that will directly influence or improve clinical practice. The purpose of applied research is to
solve problems, to make decisions, or to predict or control outcomes. Basic, or pure, research
is a scientific investigation that involves the “pursuit of knowledge for knowledge’s sake,” or
for the pleasure of learning and finding truth. The purpose of basic research is to generate and
refine theory and build constructs; thus, the findings are frequently not directly useful in
practice. However, because the findings are more theoretical in nature, they can be
generalized to various settings. Descriptive quantitative research and qualitative research
describe what exists but do not test specific interventions used in practice.

DIF: Cognitive Level: Application REF: Page 35

2. A researcher randomly assigns a large group of subjects who are hospital patients either to
receive magnesium at bedtime or not to receive magnesium at bedtime, and then measures
sleep quality and duration. What type of research is this?
a. Correlational research
b. Experimental research
c. Descriptive research
d. Quasi-experimental research
ANS: B
Control occurs when the researcher imposes “rules” to decrease the possibility of error and
thus increases the probability that the study’s findings are an accurate reflection of reality.
Descriptive and correlational studies are usually conducted with minimal control of the study
design, because subjects are examined as they exist. In experimental research, the independent
and dependent variables are highly controlled, the researcher exerts high control over the
planning and implementation of the study, and often these studies are conducted in a
laboratory setting on animals or objects. If a research study randomly assigns subjects to two
different groups, applies an intervention to one of the groups, and then measures both groups
and compares them, it is experimental design.

DIF: Cognitive Level: Analysis REF: Page 36

Use the following information to answer Questions 3 through 7:


A research study contains the following in its Introduction section: “This study was
undertaken to explore the effect of massage on total hours of sleep per 24-hour day, in persons
averaging fewer than 7 hours of sleep per night, attributable to insomnia. . . . Presumably by
increasing endorphin levels, massage seems to provide an immediate relaxation and an ability
to sleep immediately following the session, but it is unclear whether these benefits actually
extend to total sleep, despite anecdotal support. The claim that massage increases total hours
of sleep has been inadequately researched. . . . Does massage increase the total number of
hours of daily sleep? . . . It was posited that provision of daily late-morning massage would
affect total hours of sleep per 24-hour day. The study’s causational explanation was based on
the physiologic matrix of McCarthy, which includes effects of endorphins on sleep, learning
ability, pain, digestive function, and cardiac output. . . . It was taken as established fact that
massage is pleasant, that research subjects getting fewer than 7 hours of sleep per night were
sleep-deprived, and that endorphins mediated the changes observed.”

3. What is the research problem?


a. This study was undertaken to explore the effect of massage on total hours of sleep
per 24-hour day, in persons averaging fewer than 7 hours of sleep per night,
attributable to insomnia.
b. It was posited that provision of daily late-morning massage would affect total
hours of sleep per 24-hour day.
c. It was taken as established fact that massage is pleasant, that research subjects
getting fewer than 7 hours of sleep per night were sleep-deprived, and that
endorphins mediated the changes observed.
d. Presumably by increasing endorphin levels, massage seems to provide an
immediate relaxation and an ability to sleep immediately following the session, but
it is unclear whether these benefits actually extend to total sleep, despite anecdotal
support.
ANS: D
A research problem is an area of concern or phenomenon of interest about which there is a
gap in the knowledge base needed for nursing practice. The problem identifies an area of
concern or phenomenon of interest for a particular population and often indicates the concepts
to be studied.

DIF: Cognitive Level: Analysis REF: Page 39

4. What is the research framework?


a. It was taken as established fact that massage is pleasant, that research subjects
getting fewer than 7 hours of sleep per night were sleep-deprived, and that
endorphins mediated the changes observed.
b. Presumably by increasing endorphin levels, massage seems to provide an
immediate relaxation and an ability to sleep immediately following the session, but
it is unclear whether these benefits actually extend to total sleep, despite anecdotal
support.
c. It was posited that provision of daily late-morning massage would affect total
hours of sleep per 24-hour day.
d. The study’s causational explanation was based on the physiologic matrix of
McCarthy, which includes effects of endorphins on sleep, learning ability, pain,
digestive function, and cardiac output.
ANS: D
A framework is the abstract, logical structure of meaning that will guide the development of a
study and enable the researcher to link the findings to the body of nursing knowledge. In
quantitative research, the framework is often a testable midrange theory that has been
developed in nursing or in another discipline, such as psychology, physiology, or sociology.

DIF: Cognitive Level: Analysis REF: Page 41

5. What is the research assumption?


a. This study was undertaken to explore the effect of massage on total hours of sleep
per 24-hour day, in persons averaging fewer than 7 hours of sleep per night,
attributable to insomnia.
b. It was posited that provision of daily late-morning massage would affect total
hours of sleep per 24-hour day.
c. It was taken as established fact that massage is pleasant, that research subjects
getting fewer than 7 hours of sleep per night were sleep-deprived, and that
endorphins mediated the changes observed.
d. Presumably by increasing endorphin levels, massage seems to provide an
immediate relaxation and an ability to sleep immediately following the session, but
it is unclear whether these benefits actually extend to total sleep, despite anecdotal
support. The claim that massage increases total hours of sleep has been
inadequately researched.
ANS: C
Assumptions are statements that are taken for granted or that are considered true, even though
they have not been scientifically tested. Assumptions are often embedded (unrecognized) in
thinking and behavior, and uncovering them requires introspection. Sources of assumptions
include universally accepted truths, theories, previous research, and nursing practice.

DIF: Cognitive Level: Analysis REF: Page 41

6. What is the research purpose?


a. This study was undertaken to explore the effect of massage on total hours of sleep
per 24-hour day, in persons averaging fewer than 7 hours of sleep per night,
attributable to insomnia.
b. It was posited that provision of daily late-morning massage would affect total
hours of sleep per 24-hour day.
c. Presumably by increasing endorphin levels, massage seems to provide an
immediate relaxation and an ability to sleep immediately following the session, but
it is unclear whether these benefits actually extend to total sleep, despite anecdotal
support. The claim that massage increases total hours of sleep has been
inadequately researched.
d. The study’s causational explanation was based on the physiologic matrix of
McCarthy, which includes effects of endorphins on sleep, learning ability, pain,
digestive function, and cardiac output.
ANS: A
The research purpose is generated from the problem and identifies the specific focus or aim of
the study. The focus of the study might be to identify, describe, explain, or predict a solution
to a situation. The purpose often indicates the type of study to be conducted (descriptive,
correlational, quasi-experimental, or experimental) and usually includes the variables,
population, and setting for the study.

DIF: Cognitive Level: Analysis REF: Page 41

7. What is the research question?


a. This study was undertaken to explore the effect of massage on total hours of sleep
per 24-hour day, in persons averaging fewer than 7 hours of sleep per night,
attributable to insomnia.
b. It was posited that provision of daily late-morning massage would affect total
hours of sleep per 24-hour day.
c. Does massage increase the total number of hours of daily sleep?
d. Presumably by increasing endorphin levels, massage seems to provide an
immediate relaxation and an ability to sleep immediately following the session, but
it is unclear whether these benefits actually extend to total sleep, despite anecdotal
support.
ANS: C
Research objectives, questions, and hypotheses bridge the gap between the more abstractly
stated research problem and purpose and the study design and plan for data collection and
analysis. Objectives, questions, and hypotheses are narrower in focus than the research
purpose and often (1) specify only one or two research variables, (2) identify the relationship
between the variables, and (3) indicate the population to be studied. A research question is a
concise, interrogative statement that is worded in the present tense and includes one or more
variables (or concepts).

DIF: Cognitive Level: Analysis REF: Page 39

8. A researcher conducting a study to examine linkages among age, gender, driver’s license
suspension, and zip code poverty, educational level, and income, sourced from the records of
the State Department of Motor Vehicles, is using which of the following types of research?
a. Descriptive research
b. Correlational research
c. Problem solving
d. Triangulation
ANS: B
Through descriptive research, concepts are described and relationships are identified but not
examined. A researcher conducting a study to examine linear relationships between two or
more variables is using the quantitative research process for correlational research.
In descriptive and correlational studies, no treatment is administered, so the study design
centers on describing variables, examining relationships, and improving the precision of
measurement. In descriptive research concepts are explored and phenomena are described in
real-life situations. This approach is used to generate new knowledge about concepts or topics
about which limited or no research has been conducted. In correlational research linear
relationships between two or more variables are explored and the strength between variables
is quantified.
DIF: Cognitive Level: Application REF: Page 49

9. A student completes her master’s thesis on correlates of depression in retired airline pilots,
and it is shelved in the library. Has this student communicated her research findings?
a. Yes, because the thesis is in the library and can be accessed.
b. No, because the findings have not been made available to persons who will utilize
them.
c. Yes, because the students in this particular master’s program often discuss their
work in progress.
d. No, because if the findings do not appear in print in a nursing journal, they have
not been communicated.
ANS: B
Research is not considered complete until the findings have been communicated.
Communicating research findings involves developing and disseminating a research report to
appropriate audiences; the research report is disseminated through presentations and
publications. Analyzing data, drawing conclusions, and writing a report of the findings are
essential steps in conducting research—but they do not complete the process.

DIF: Cognitive Level: Analysis REF: Page 49

10. Hospital nurses are observed in order to determine exactly how long nurses swab IV ports
with alcohol. Because they are being observed, they “scrub the hub” longer than they
ordinarily would have. This is an example of what concept relevant to quantitative research?
a. Bias
b. Control
c. Inaccurate operationalization of variables
d. Hawthorne effect
ANS: D
Subjects’ knowledge of a study could influence their behavior and possibly alter the research
outcomes. This threatens the validity or accuracy of the study design. An example of this type
of threat to design validity is the Hawthorne effect, which was identified during the classic
experiment at the Hawthorne plant of the Western Electric Company during the 1920s and
1930s. The employees at this plant exhibited a particular psychological response when they
became research participants: they changed their behavior simply because they were subjects
in a study, not because of the research treatment.

DIF: Cognitive Level: Analysis REF: Page 38

11. A researcher studies the effect upon dental caries formation of a year-long regimen of daily
rinsing with a particularly noxious-flavored oral solution, only to discover than 285 of the 300
subjects in the study have withdrawn from it by the end of the first month. Which step in the
research process was not properly undertaken?
a. Defining the purpose
b. Conducting the literature review
c. Selecting study variables
d. Performing a pilot study
ANS: D
A pilot study is commonly defined as a smaller version of a proposed study conducted to
refine the methodology. It is developed much like the proposed study, using similar subjects,
the same setting, the same treatment, and the same data collection and analysis techniques.
Some of the reasons for conducting pilot studies are to determine feasibility, to develop a
treatment or intervention, to develop an implementation protocol, to identify problems with
the design, to evaluate the sampling method, to examine instrument reliability or validity, to
refine instruments, to refine plans for data collection and analysis, to provide research
experience, and to evaluate data analysis techniques.

DIF: Cognitive Level: Application REF: Page 46

12. A researcher studies the effect of three one-hour counseling sessions on eliminating bullying
behaviors in teenagers. For the dependent variable, the researcher selects the outcome of being
reported to the principal’s office in the two weeks following the sessions. The results of the
research are dismissed by reviewers as meaningless, severely limiting generalization. What is
the problem here?
a. Theoretical limitations
b. Not enough independent variables
c. Methodological limitations
d. Insufficient sample size
ANS: A
Limitations are restrictions or problems in a study that may decrease the generalizability of
the findings. Study limitations often include a combination of theoretical and methodological
weaknesses. Theoretical weaknesses in a study might include poorly developed study
framework and unclear conceptual definitions of variables. The limited conceptual definitions
of the variables might decrease the operationalization or measurement of the study variables.
This is the case here, since a two-week measuring period is sufficient only for purposes of
suppressing a behavior, not changing it. Methodological limitations result from factors such as
nonrepresentative samples, weak designs, single setting, limited control over treatment
(intervention), instruments with limited reliability and validity, limited control over data
collection, and improper use of statistical analyses. These study limitations can limit the
credibility of the findings and conclusions and restrict the population to which the findings
can be generalized.

DIF: Cognitive Level: Analysis REF: Page 48

13. Which is the best statement that differentiates between the sizes of samples and populations?
a. A sample has a maximum size; a population does not.
b. A sample cannot be the same size as a population.
c. If a person is a member of a population, he or she is a member of the sample
chosen from that population.
d. A population is usually larger than a sample.
ANS: D
The population is all the elements (individuals, objects, or substances) that meet certain
criteria for inclusion in a given universe. The researcher must determine which population is
accessible and can be best represented by the study sample. A sample is a subset of the
population that is selected for a particular study. Being a subset, the sample is either smaller
than the population or, very occasionally, equal in size to it; it cannot be larger.
DIF: Cognitive Level: Analysis REF: Page 44

14. What does a quantitative research instrument measure?


a. The level of measurement
b. A statistical test
c. Itself, for validity
d. A study variable
ANS: D
When conducting a quantitative study, the researcher attempts to use the most precise
instruments available to measure the study variables.

DIF: Cognitive Level: Comprehension REF: Page 37

15. Which is the highest form of measurement?


a. Interval
b. Nominal
c. Ordinal
d. Ratio

ANS: D
An instrument is selected to measure a specific variable in a study. Data generated with an
instrument are at the nominal, ordinal, interval, or ratio level of measurement. The level of
measurement, with nominal being the lowest form of measurement and ratio being the
highest, determines the type of statistical analyses that you can perform on the data.

DIF: Cognitive Level: Comprehension REF: Page 45

16. The researcher believes that adults can remember details about the first time they were taken
on a camping trip, as 7-year-olds, and that the experiences of a first camping trip are life-
altering. What is a research term for these beliefs?
a. Applications
b. Assumptions
c. Limitations
d. Variables
ANS: B
Assumptions are statements that are taken for granted or are considered true, even though they
have not been scientifically tested. Assumptions are often embedded (unrecognized) in
thinking and behavior, and uncovering them requires introspection. Sources of assumptions
include universally accepted truths (e.g., all humans are rational beings), theories, previous
research, and nursing practice. In studies, assumptions are embedded in the philosophical base
of the framework, study design, and interpretation of findings. Theories and instruments are
developed on the basis of assumptions that the researcher may or may not recognize. These
assumptions influence the development and implementation of the research process. Since
researchers’ assumptions influence the logic of the study, their recognition leads to more
rigorous study development.

DIF: Cognitive Level: Application REF: Page 41


17. Which of the following items is different when comparing probability sampling and
nonprobability sampling?
a. The type of descriptive statistics applied to the sample
b. The size of the sample
c. The relative chance of being selected as a study participant
d. Whether or not the findings can be generalized
ANS: C
Sampling is a process of selecting subjects, events, behaviors, or elements for participation in
a study. Random sampling methods usually provide a sample that is representative of a
population, because each member of the population has a probability greater than zero of
being selected for a study. This is not true of nonrandom sampling methods, in which not
every member of the population has an opportunity for selection to the sample. Descriptive
statistics applied to the sample are identical. The size of the sample doesn’t vary depending on
type of sample chosen. Generalization of the findings is possible under either condition.

DIF: Cognitive Level: Analysis REF: Page 37

18. A correlational researcher reports that the strength of the relationship between X and Y is near
0 (r = 10.03). What does this mean, relative to prediction?
a. If X is present, Y is only somewhat likely to be present.
b. If Y is absent, X will also be absent.
c. If X is present, there is no guarantee at all that Y will be present.
d. If Y is absent, X will always be present.

ANS: C
Correlational research examines a linear relationship between two or more variables and
determines the type (positive or negative) and degree (strength) of the relationship. The
strength of a relationship varies from –1 (perfect negative correlation) to +1 (perfect positive
correlation), with 0 indicating no relationship. The positive relationship indicates that the
variables vary together—that is, the two variables either increase or decrease together. The
negative or inverse relationship indicates that the variables vary in opposite directions; thus,
as one variable increases, the other variable decreases.

DIF: Cognitive Level: Application REF: Page 49

MULTIPLE RESPONSE

1. How is a researcher who exemplifies rigor similar to the best technical nurse on her shift in a
cardiovascular intensive care unit? (Select all that apply.)
a. They are both rigid and inflexible in details such as timelines and doing things the
way they want to do them.
b. They are both aggressive in acquiring and recording data.
c. They both strive for excellence.
d. They both communicate well with others.
e. They both are disciplined in the way they conduct their jobs.
f. They both are passionate about accuracy and attending to details.
ANS: C, E, F
Rigor is striving for excellence in research and involves discipline, scrupulous adherence to
detail, and strict accuracy. A rigorous quantitative researcher constantly strives for more
precise measurement methods, structured treatment, representative samples, and tightly
controlled study designs. Characteristics valued in these researchers include critical
examination of reasoning and attention to precision.

DIF: Cognitive Level: Synthesis REF: Page 49

2. A researcher undertakes a research study on the danger of bears in Yosemite Valley. What
determines the researcher’s selection of a research design? (Select all that apply.)
a. The mentor the researcher chooses to support the study
b. Whether or not the researcher intends to generalize the findings
c. The researcher’s expertise and comfort with the research process chosen
d. Whether the National Park Systems are funding the research
e. The study purpose and its anticipated outcomes
f. The body of research already present on bear danger
ANS: B, C, E, F
A research design is a blueprint for maximizing control over factors that could interfere with a
study’s desired outcome. The choice of research design depends on the researcher’s expertise,
the problem and purpose for the study, and the desire to generalize the findings. This means
that other research in the area must be taken into consideration as well.

DIF: Cognitive Level: Application REF: Page 43

3. A researcher performs a study of how many nurses are assigned to a nursing floor on the basis
of total square feet of the unit, correlating this with injury, fatigue, patient assignment, patient
acuity, and length of employment. Which of the following are true regarding the type of
quantitative data analysis used by the researcher? (Select all that apply.)
a. The type of quantitative data analysis guides the study objectives and hypotheses.
b. The type of quantitative data analysis is determined by the level of measurement of
data.
c. The type of quantitative data analysis determines the research design.
d. The type of quantitative data analysis should include some sort of numerical
analysis.
e. The kind of data that will be collected is determined by the type of analysis
chosen.
ANS: B, D
Planning data analysis is the final step before the study is implemented. The analysis plan is
based on (1) the research objectives, questions, or hypotheses; (2) the data to be collected; (3)
research design; (4) researchers’ expertise; and (5) availability of computer resources. Several
statistical analysis techniques are available to describe the sample, examine relationships, or
determine significant differences within studies. Most researchers consult a statistician for
assistance in developing an analysis plan.

DIF: Cognitive Level: Application REF: Page 45

4. A researcher conducts a pilot study before the main study is conducted. Why might the
researcher choose to do this? (Select all that apply.)
a. The researcher has no idea whether subjects will complete the various phases of
the study.
b. The researcher needs to know how much of a change will occur in the dependent
variable, so that sample size can be determined.
c. The researcher isn’t sure whether the fourth phase of the study is really necessary.
d. The study site is a new one, and the researcher wants to find out whether it’s
suitable for this kind of research.
e. The researcher has insufficient funding for a large study.
f. The researcher isn’t sure whether the tool he or she is using to measure the
dependent variable will be practical.
ANS: A, B, C, D, F
A pilot study is commonly defined as a smaller version of a proposed study conducted to
refine the methodology. It is developed much like the proposed study, using similar subjects,
the same setting, the same treatment, and the same data collection and analysis techniques.
Some of the reasons for conducting pilot studies are to determine feasibility, to develop a
treatment or intervention, to develop an implementation protocol, to identify problems with
the design, to evaluate the sampling method, to examine instrument reliability or validity, to
refine instruments, to refine plans for data collection and analysis, to provide research
experience, and to evaluate data analysis techniques.

DIF: Cognitive Level: Synthesis REF: Page 46

5. What is applied research? (Select all that apply.)


a. Research that has been applied in the past to practice
b. Research directly useful in clinical practice
c. Research usually conducted in the setting in which it will be applied
d. Research that is conducted with paid volunteers
e. Research conducted to generate knowledge that will directly and indirectly
influence or improve clinical practice
ANS: B, C, E
Basic, or pure, research is a scientific investigation that involves the pursuit of “knowledge for
knowledge’s sake,” or for the pleasure of learning and finding truth. The purpose of basic
research is to generate and refine theory and build constructs; thus, the findings are frequently
not directly useful in practice. Applied, or practical, research is a scientific investigation
conducted to generate knowledge that will directly influence or improve clinical practice. The
purpose of applied research is to solve problems, to make decisions, or to predict or control
outcomes in real-life practice situations.

DIF: Cognitive Level: Analysis REF: Page 53

6. A marketing researcher reviews the month’s sales slips for a convenience store and compares
them with restocking orders, in order to determine which products are being stolen from the
shelves. This study has little control. Why is this the case? (Select all that apply.)
a. The researcher has no control over whether people choose to shoplift.
b. There is no control for extraneous variables.
c. No variables are manipulated.
d. The design is descriptive or correlational; as compared with other types of
research, control is low.
e. The data collected were actually generated by other people and may be erroneous.
ANS: B, C, D, E
Control occurs when the researcher imposes “rules” to decrease the possibility of error and
thus increase the probability that the study’s findings are an accurate reflection of reality.
Through control, the researcher can reduce the influence or confounding effect of extraneous
variables on the study variables. Quantitative research requires varying degrees of control,
ranging from minimal control to high control of study design. Descriptive and correlational
studies are usually conducted with minimal or partial control of the study design.
Correlational research often has more control of its design than does descriptive research.
Quasi-experimental studies are usually conducted with moderate control of study design.
Experimental studies are highly controlled.

DIF: Cognitive Level: Application REF: Page 36

7. In a given research study, the findings reveal that as A increases, B also increases, that the
relationship is linear, and that the strength of the relationship is 0.78. What type of
relationship is this? (Select all that apply.)
a. Positive
b. Negative
c. Inverse
d. None
e. Causational
f. Correlational
ANS: A, F
Correlational research examines a linear relationship between two or more variables and
determines the type (positive or negative) and degree (strength) of the relationship. The
strength of a relationship varies from –1 (perfect negative correlation) to +1 (perfect positive
correlation), with 0 indicating no relationship. The positive relationship indicates that the
variables vary together—that is, the two variables either increase or decrease together. The
negative or inverse relationship indicates that the variables vary in opposite directions; thus,
as one variable increases, the other variable decreases.

DIF: Cognitive Level: Application REF: Page 49

8. A researcher selects a quantitative experimental research design. For what reasons does the
researcher select this particular design? (Select all that apply.)
a. To generate a theory
b. To answer a research question
c. To determine which of several causes is the true one
d. To prove a theory
e. To disprove a hypothesis
f. To determine the strength of the relationship between the independent variable and
the dependent variable
ANS: B, F
A research design is a blueprint for maximizing control over factors that could interfere with a
study’s desired outcome. The choice of research design depends on the researcher’s expertise,
the problem and purpose for the study, and the desire to generalize the findings. This means
that other research in the area must be taken into consideration as well.

DIF: Cognitive Level: Application REF: Page 43


9. The director of a major hospital complex conducts a study to discover the types of critical
incidents that have occurred in this hospital and its sister hospital over the past five years. She
makes a list of every critical incident that has occurred over this period. Choose the true
statements about this list. (Select all that apply.)
a. The list is the dependent variable.
b. The list represents the hospital director’s assumptions.
c. The list is an extraneous variable.
d. The list represents the sample.
e. If the two hospitals have been in operation only five years, the list represents the
population.
ANS: A
The population is all the elements (individuals, objects, or substances) that meet certain
criteria for inclusion in a given universe. The researcher must determine which population is
accessible and can be best represented by the study sample. A sample is a subset of the
population that is selected for a particular study. Being a subset, the sample is either smaller
than the population or, very occasionally, equal in size to it; it cannot be larger.

DIF: Cognitive Level: Application REF: Page

MULTIPLE CHOICE

1. What does the “grounded” in grounded theory mean?


a. Small pieces of data are “ground up” in the analysis process.
b. The theory that emerges is “grounded” in real-world data.
c. No theory is groundless.
d. All data must be “on the ground” and written out fully.
ANS: B
Grounded theory research is an inductive research technique developed by Glaser and Strauss
through their study of the experience of dying. The method’s name means the findings are
grounded in the concrete world as experienced by the participants and are interpreted at a
more abstract theoretical level.

DIF: Cognitive Level: Analysis REF: Page 62

2. An ethnographic researcher plans to study organizations and how they promote or suppress
individual effort. What type of ethnography will the researcher select?
a. Classical ethnography
b. Systematic ethnography
c. Interpretive ethnography
d. Critical ethnography

ANS: B
Four schools of thought within ethnography have emerged from different philosophical
perspectives and these include classical, systematic, interpretive, and critical ethnography.
Classical ethnography seeks to provide a comprehensive holistic description of a culture. In
contrast, systematic ethnography explores and describes the structures of the culture with an
increased focus on groups, patterns of social interaction, organizations, and institutions.
Interpretive ethnography has as its goal understanding the values and thinking that result in
the behaviors and symbols of the people being studied. Critical ethnography has a political
purpose of relieving oppression and empowering a group of people to take action on their own
behalf. It is not ethnography’s focus to construct theories.

DIF: Cognitive Level: Application REF: Page 64

3. Why is the Sunshine Model of ethnonursing more specific to health than other ethnography
models?
a. It was created by a nurse.
b. It values the point of view of the individual.
c. It focuses on factors that impact health.
d. It explains how various levels of culture interact.
ANS: C
Madeline Leininger (1970) brought ethnography into nursing science by writing the first book
linking nursing with anthropology. Leininger was first a nurse and then earned her doctoral
degree in anthropology. In the 1950s, she began developing a framework for culture care that
became the Sunshine Model (Clarke, McFarland, Andrews, & Leininger, 2009). The Sunshine
Model identifies factors that affect health and illness, such as religion, income, kinship,
education, values, and beliefs. Chapter 7 contains more information about the Theory of
Culture Care developed by Leininger, so this section focuses on the qualitative method that
she developed to be consistent with ethnonursing. Multiple levels of factors affect the culture
and, consequently, the care expressions of the people. A person who is a member of the only
Vietnamese family in a small rural community in Georgia may have different care practices
than if he or she were living in New York City in a predominantly Vietnamese community.

DIF: Cognitive Level: Analysis REF: Page 65

4. A researcher investigates the fact that women with chronic pain are more apt to be treated for
depression than are men with chronic pain. Which qualitative strategy will most likely be used
to study this topic?
a. Grounded theory
b. Exploratory-descriptive qualitative research
c. Phenomenology
d. Critical research
ANS: D
Critical ethnography has a political purpose of relieving oppression and empowering a group
of people to take action on their own behalf.

DIF: Cognitive Level: Analysis REF: Page 64

MULTIPLE RESPONSE
1. A research study about holiday celebrations is based on a philosophy or philosophical
perspective. In the analysis, the authors state that they reflected upon the data for several
weeks, reading and re-reading interviews, in order to capture their meaning. Aside from
descriptive statistics addressing the sample, the results are all presented in narrative form.
Which of the following statements are true? (Select all that apply.)
a. The philosophy for the study is logical positivism.
b. The sample size was decided upon using power analysis.
c. In this method, meaning emerges from the data.
d. The data analysis process seems to be inductive.
e. The method was shaped by the authors’ philosophical perspectives.
ANS: C, D, E
Quantitative studies are based primarily on the philosophy of logical positivism that values
logic, empirical data, and tightly controlled methods. Power analysis is a quantitative method
of setting the sample size. Inductive thinking involves perceptually putting insights and pieces
of information together and identifying abstract themes or working from the bottom up. From
this inductive process, meanings emerge. In qualitative research, the philosophy directs the
research questions and the collection and interpretation of the data.

DIF: Cognitive Level: Application REF: Page 57

2. Which statements best describes the differences between Heideggerian and Husserlian
phenomenology? (Select all that apply.)
a. Husserl proposed that the researcher could identify and set aside his or her own
private attitudes and opinions before data analysis.
b. Heidegger postulated that a person interacted with the world only through his or
her physical body.
c. Heideggerians believe that the past has no influence on present thought.
d. Heideggerian phenomenologists posit that the person is situated in a specific
context and time that shape his or her experiences, paradoxically freeing and
constraining the person’s ability to establish meanings through language, culture,
history, purposes, and values.
e. Husserl developed his ideas as a method for understanding and avoiding conflict
between psychology and the basic sciences.
ANS: A, B, D, E
Heideggerian phenomenologists believe that the person is a self within a body; the person is
referred to as embodied. Husserlian phenomenologists believe that although self and world are
mutually shaping, it is possible to separate oneself from one’s beliefs or set aside one’s beliefs
to see the world firsthand in a naive way. Setting aside one’s beliefs during qualitative
research is called bracketing. Heideggerian phenomenologists posit that the person is situated
in a specific context and time that shape his or her experiences, paradoxically freeing and
constraining the person’s ability to establish meanings through language, culture, history,
purposes, and values. Husserl developed his ideas about phenomena in an effort to resolve the
conflict in thought between human sciences (primarily psychology) and the basic sciences
(such as physics).

DIF: Cognitive Level: Analysis REF: Page 60

3. What is the major contribution of historical nursing research? (Select all that apply.)
a. It explains how to avoid pitfalls of the past.
b. It allows us to explain the world of today through the lens of yesterday.
c. It identifies recurrent social patterns.
d. It tells the story of where we have been as a profession.
e. It provides the dates of important events.
ANS: B, D
Historical research examines events of the past. Historians describe events in the context of
time, social structures, concurrent events, and key individuals. These descriptions can increase
understanding and raise awareness of the forces shaping current events. Historical nursing
research can do the same for the profession and its role in society. Nurse researchers using
historical methods have examined the events and people that shaped health in different
settings and countries as well as nursing as a profession.

DIF: Cognitive Level: Application REF: Page 68

4. Which of the following might be the focus of historical nursing research? (Select all that
apply.)
a. The evolution of the role of the physician in the 20th century
b. Diseases that resulted in significant mortality in the 18th century
c. Patterns of nursing staffing in years of shortage prior to 1980
d. A person or persons who have contributed to the profession of nursing
e. Social patterns that have fostered or squelched nurses’ developing autonomy
ANS: C, D, E
Historical research examines events of the past. Historians describe events in the context of
time, social structures, concurrent events, and key individuals. These descriptions can increase
understanding and raise awareness of the forces shaping current events. Historical nursing
research can do the same for the profession and its role in society. Nurse researchers using
historical methods have examined the events and people that shaped health in different
settings and countries as well as nursing as a profession.

DIF: Cognitive Level: Application REF: Page 68

5. What is exploratory-descriptive qualitative research? (Select all that apply.)


a. Research that is clearly qualitative but that does not espouse any distinct
methodology
b. Quantitative research that contains descriptions
c. Research that contains elements of at least two other types of qualitative research
d. Mixed methods research
e. A non-method
ANS: A, E
Qualitative nurse researchers have conducted studies with the purpose of exploring and
describing a topic of interest but, at times, have not identified a specific qualitative
methodology. Qualitative descriptive research is a legitimate method of research that may be
the appropriate “label” for studies that have no clearly specified method or when the method
was specified but ended with “a comprehensive summary of an event in the everyday terms of
these events.” Labeling a study as a specific type (grounded theory, phenomenology, or
ethnographic) implies fixed categories of research with distinct boundaries, but the boundaries
between methods are more appropriately viewed as permeable. Although the studies result in
descriptions and could be labeled as descriptive qualitative studies, most of the researchers are
in the exploratory stage of studying the subject of interest.

DIF: Cognitive Level: Analysis REF: Page 66

6. What are the focal points of the four schools of thought within ethnography? (Select all that
apply.)
a. To relieve oppression and empower a group to take action on its own behalf
b. To investigate cultural structures, focusing on groups and their social patterns
c. To construct theories that explain cultural interactions
d. To understand values and thinking that collectively result in behaviors and
symbols of the individuals within a culture
e. To provide a comprehensive holistic description of a culture
ANS: A, B, D, E
Four schools of thought within ethnography have emerged from different philosophical
perspectives and these include classical, systematic, interpretive, and critical ethnography.
Classical ethnography seeks to provide a comprehensive holistic description of a culture. In
contrast, systematic ethnography explores and describes the structures of the culture with an
increased focus on groups, patterns of social interaction, organizations, and institutions.
Interpretive ethnography has as its goal understanding the values and thinking that result in
the behaviors and symbols of the people being studied. Critical ethnography has a political
purpose of relieving oppression and empowering a group of people to take action on their own
behalf. It is not ethnography’s focus to construct theories.

DIF: Cognitive Level: Synthesis REF: Page 64

7. Which of the following are the general purposes of ethnographic research? (Select all that
apply.)
a. To describe events within a culture in the distant past
b. To describe a culture
c. To construct theories that explain cultural interactions
d. To measure the effect of living conditions on human care, health, and nursing
process
e. To explore meanings of social actions against a cultural backdrop
ANS: B, E
Ethnographic research provides a framework for studying cultures. The word ethnography is
derived by combining the Greek roots of “ethno” (folk or people) and “graphy” (picture or
portrait). Ethnographies are the written reports of a culture from the perspectives of insiders.
These reports were initially the products of anthropologists who studied primitive, foreign, or
remote cultures. Now, however, a number of other disciplines, including social psychology,
sociology, political science, education, and nursing, promote cultural research.
Anthropologists seek to understand people: their ways of living, believing, acquiring
information, transforming knowledge, and socializing the next generation. Studying a culture
begins with the philosophical values of respecting, appreciating, and seeking to preserve the
values and ways of life of the culture. The philosophical bases of ethnography are naturalism
and respect for others. The purpose of anthropological research is to describe a culture and
explore “the meanings of social actions within cultures.”

DIF: Cognitive Level: Synthesis REF: Page 63

8. Which of the following are the general purposes of phenomenological research? (Select all
that apply.)
a. To generate theory
b. To describe the lived experience
c. To observe and document interactions within an existent culture
d. To determine the meaning that an experience has for the individual
e. To describe the single reality expressed by a group of participants
ANS: B, D
The purpose of phenomenological research is to describe experiences (or phenomena) as they
are lived—in phenomenological terms, to capture the “lived experience” of study participants.
During the process of data collection, the meaning the participants attach to their experience is
revealed. All phenomenologists agree that there is not a single reality: each individual has his
or her own reality. While phenomenology can be used indirectly in the development of a
theory, grounded theory is the method intended to generate theory. Observing and
documenting interactions within a culture describes ethnographic research.

DIF: Cognitive Level: Comprehension REF: Page 60

9. A certain qualitative method takes the position that there is no single reality. Because of this,
the reality experienced by each participant is unique. Because experience is subjective, the
experienced reality is reality. The method does not perform reality checks in order to
determine whether a participant’s story is “true” or not. What is this qualitative method?
(Select all that apply.)
a. Husserlian phenomenology
b. Ethnography
c. Historicism
d. Heideggerian phenomenology
e. Grounded theory
ANS: A, D
The purpose of phenomenological research is to describe experiences (or phenomena) as they
are lived—in phenomenological terms, to capture the “lived experience” of study participants.
All phenomenologists agree that there is not a single reality; each individual has his or her
own reality. Reality is considered subjective, and as a result, unique to the individual.
DIF: Cognitive Level: Analysis REF: Page 60

10. What are the general truths of symbolic interaction theory, as utilized in grounded theory
research? (Select all that apply.)
a. Perceptions of one’s interactions with others shape one’s self-view.
b. Perceptions of one’s interactions with others shape subsequent interactions.
c. A person is “embodied” and experiences the world within that body.
d. The culture determines behavior; the persons comprise the culture.
e. Persons within a social structure share symbols that have meaning for them.
ANS: A, B, E
Heideggerian phenomenologists believe that the person is a self within a body, which is
referred to as a person being embodied. Symbolic interaction theory explores how perceptions
of interactions with others shape one’s view of self and subsequent interactions. In social life,
groups share meanings. They communicate these shared meanings to others through
socialization processes. Ethnography does not require travel to another country or region;
however, it requires spending considerable time in the setting observing and gathering data.

DIF: Cognitive Level: Analysis REF: Page 60

11. Which of the following are the characteristics of grounded theory research? (Select all that
apply.)
a. It focuses on experiences and processes, against the backdrop of society.
b. It scrutinizes phenomena, past the capabilities of quantitative research.
c. It always develops theory.
d. It provides a cohesive description of a phenomenon, fostering understanding.
e. It is able to be used effectively in a considerable variety of settings.
ANS: A, B, D, E
Grounded theory research is an inductive research technique developed by Glaser and Strauss
through their study of the experience of dying. The method’s name means the findings are
grounded in the concrete world as experienced by the participants and are interpreted at a
more abstract theoretical level. The desired outcome of grounded theory studies is a middle
range or substantive theory. Grounded theory researchers have contributed to our
understanding of the patient experience across a wide range of settings. Grounded theory
research examines experiences and processes with a breadth and depth not usually possible
with quantitative research. The reader can intuitively verify these findings through her or his
own experiences. The contribution to nursing science is that a clear, cohesive description of
the phenomenon allows greater understanding.

DIF: Cognitive Level: Synthes

MULTIPLE CHOICE

1. A researcher has conducted 9 clinical studies, some quantitative and others qualitative, all of
which focus on depression’s relationship to perceived abandonment. “Depression’s
relationship to perceived abandonment” is an example of which of the following?
a. Research problem
b. Research topic
c. Research purpose
d. Problem statement
ANS: B
Research topics are concepts, phenomena of interest, or broad problem areas that researchers
can focus on to enhance evidence-based nursing. A research problem is an area of concern
where there is a gap in the knowledge base needed for nursing practice. Research topics
contain numerous potential research problems, and each problem provides the basis for
developing many research purposes. The problem statement identifies the specific gap in the
knowledge needed for practice. A nursing situation often includes a variety of research topics
or concepts.

DIF: Cognitive Level: Application REF: Page 73

2. Why is replicating a research study essential for knowledge development?


a. Each time a study is replicated, its probability of error decreases.
b. Reproducing a study decreases theoretical knowledge, increasing real knowledge.
c. Replication helps confirm that the initial results were not reached in error.
d. Replication studies represent the majority of published nursing literature.

ANS: C
Replication involves reproducing or repeating a study to determine if similar findings will be
obtained. Replication is essential for knowledge development because it (1) establishes the
credibility of the findings, (2) extends the generalizability of the findings over a range of
instances and contexts, (3) reduces the number of type I and type II errors, (4) corrects the
limitations in studies’ methodologies, (5) supports theory development, and (6) lessens the
acceptance of erroneous results. Some researchers replicate studies because they agree with
the findings and wonder if the findings will hold up in different settings with different subjects
over time. Others want to challenge the findings or interpretations of prior investigators.
However, the number of nursing studies replicated continues to be limited.

DIF: Cognitive Level: Synthesis REF: Page 77

3. A nurse researcher working in a subacute orthopedic hospital floor. She notes that her elders
with knee replacements sleep as many as 16 hours a day, waking only for physical therapy
and meals, but she also notices that those with many visitors sleep fewer hours and seem to
experience more pain. She wonders whether sleep in elders after knee replacement prevents
pain, or whether elders select the coping strategy of sleeping more, in response to pain, and
begins to attempt to identify the relationship between the two. A literature search reveals only
three descriptive studies on this topic, one quantitative and two qualitative. What is “the
relationship between elders’ hours of sleep following knee replacement and its relationship
with report of pain”?
a. The research aim
b. The research purpose
c. The research problem
d. The research topic
ANS: C
Research topics are concepts, phenomena of interest, or broad problem areas that researchers
can focus on to enhance evidence-based nursing. A research problem is an area of concern
where there is a gap in the knowledge base needed for nursing practice. Research topics
contain numerous potential research problems, and each problem provides the basis for
developing many research purposes.

DIF: Cognitive Level: Synthesis REF: Page 73

4. A master’s student who works in cardiothoracic ICU reads a 20-year-old nursing research
study; the findings document use of much larger per-kilogram amounts of opioids and
anxiolytics postoperatively in adults with open-heart surgery, as opposed to children with
open-heart surgery. The student strongly suspects that modern hospitals medicate children and
adults more or less the same, on a per-kilogram basis. She decides to replicate the original
research in her hospital. What type of replication is this?
a. Exact replication
b. Concurrent replication
c. Systematic replication
d. Approximate replication
ANS: D
Replication involves reproducing or repeating a study to determine whether similar findings
will be obtained. Four different types of replication are important in generating sound
scientific knowledge for nursing: (1) exact, (2) approximate, (3) concurrent, and (4)
systematic extension. An exact, or identical, replication involves duplicating the initial
researcher’s study to confirm the original findings. All conditions of the original study must
be maintained. An approximate, or operational, replication involves repeating the original
study under similar conditions, following the methods as closely as possible. A concurrent, or
internal, replication involves the collection of data for the original study and its simultaneous
replication to provide a check of the reliability of the original study. A systematic extension or
constructive replication is done under distinctly new conditions.

DIF: Cognitive Level: Application REF: Page 77

5. A research study contains the question, “Can the application of twice-daily cortisone in the
period from 6 to 10 weeks postoperatively produce significantly increased range of motion in
50- to 60-year-old rotator-cuff repair patients at the six-month mark?” The study is _____
research.
a. Ethnographic
b. Historical
c. Experimental
d. Basic
ANS: C
Experimental studies are conducted in highly controlled settings and under highly controlled
conditions to determine the effect of one or more independent variables on one or more
dependent variables. An experimental research question takes the form of, “Does provision of
increased A have an effect on B?” where the study variables are A and B, the relationship is a
change in A and its effect on B, and the question ends with a question mark.

DIF: Cognitive Level: Synthesis REF: Page 87


6. A knowledge gap is identified by a nurse researcher. Which of the following may NOT
necessarily represent a knowledge gap?
a. A literature search that shows that no quantitative research, but only qualitative
research, exists in the area.
b. The National Institutes of Health (NIH) identify the area as one of research
priority.
c. The phenomenon of interest is a new disease, just identified in Phoenix, Arizona.
d. There is one study examining a new intervention, but the research has not yet been
replicated.
ANS: A
The lack of quantitative research may mean only that the phenomenon or idea of the research
is not suited for quantitative research, since some concepts are suitable only for qualitative
enquiry. By questioning and reviewing the literature, researchers begin to recognize a specific
area of concern and the knowledge gap that surrounds it. The knowledge gap, or what is not
known about this clinical problem, determines the complexity and number of studies needed
to generate essential knowledge for nursing practice. Since 1975, expert researchers, specialty
groups, professional organizations, and funding agencies have identified nursing research
priorities.

DIF: Cognitive Level: Application REF: Page 87

7. Which of the following represents a concise, clear statement of the specific goal or aim of a
research study?
a. The research problem was identified as the general area of ignorance surrounding
the causation and diagnosis of meningococcal meningitis, especially among
soldiers in uniform, and this constituted a significant gap in the literature.
b. The purpose of the project, then, was to define changes in the variable of
hypertension across time, with the four most prevalent treatment modalities
prescribed by primary care physicians in the greater Chicago area.
c. The phenomenon of depression experienced by the primary schoolchild was
focused upon in this study. It is an under-researched topic, especially since recent
research has identified correlational links with childhood obesity, especially
Hispanic and African-American populations.
d. Despite the fact that studying the disease, especially from standpoints of
prevention and early detection, is costly and time-consuming, it represents a clear
priority, since treatment itself is remarkably costly and the afflicted are not likely
to experience even short remissions without early intervention.
ANS: B
The research purpose is a concise, clear statement of the specific goal or aim of the study that
is generated from the research problem. A research problem is an area of concern where there
is a gap in the knowledge base needed for nursing practice. Research topics are concepts,
phenomena of interest, or broad problem areas that researchers can focus on to enhance
evidence-based nursing. One source of research problems is research priorities identified by
funding agencies and specialty groups.

DIF: Cognitive Level: Comprehension REF: Page 74

8. What is the relationship between a research topic and a research problem?


a. The purpose is the most general statement; the research topic is the most specific
to the research itself.
b. Research topics contain numerous potential research problems, and each problem
provides the basis for developing many purposes.
c. The research topic and the research problem are identical.
d. The research topic specifies setting and population, but the problem does not.
ANS: A
Research topics are concepts, phenomena of interest, or broad problem areas that researchers
can focus on to enhance evidence-based nursing. Research topics contain numerous potential
research problems, and each problem provides the basis for developing many purposes. Thus,
the identification of a relevant research topic and a challenging, significant problem can
facilitate the development of numerous study purposes to direct a lifetime program of
research.

DIF: Cognitive Level: Analysis REF: Page 73

9. Which of the following is the practicing nurse’s most important source of researchable
problems?
a. The nurse’s own clinical practice
b. Review of the literature
c. Nursing theories
d. Administrative mandates to conduct clinical research on every hospital unit
ANS: A
The practice of nursing must be based on knowledge or evidence generated through research.
Thus, clinical practice is an extremely important source for research problems. Problems can
evolve from clinical observations. A review of patient records, treatment plans, and procedure
manuals might reveal concerns or raise questions about practice that could be the basis for
research problems.

DIF: Cognitive Level: Analysis REF: Page 75

MULTIPLE RESPONSE

1. Which of the following are considered evidence-generating? (Select all that apply.)
a. Replication of previous research
b. Identification of research topics, followed by basic research
c. Applied research studies that examine clinical response to interventions
d. Reviews of the literature
e. Qualitative research examining responses to diagnosis
ANS: A, B, C, E
Research topics are concepts, phenomena of interest, or broad problem areas that researchers
can focus on to enhance evidence-based nursing. However, the lack of replication studies
severely limits the generation of sound research findings needed for evidence-based practice
in nursing. Basic, or pure, research is a scientific investigation that involves the pursuit of
“knowledge for knowledge’s sake,” or for the pleasure of learning and finding truth. The
purpose of basic research is to generate and refine theory and build constructs; thus, the
findings are frequently not directly useful in practice. Replication of previously conducted
research is essential for knowledge development. By questioning and reviewing the literature,
researchers begin to recognize a specific area of concern and the knowledge gap that
surrounds it; however, review of the literature does not generate knowledge—it reviews
previous knowledge. Questions focusing on investigating new techniques to improve existing
skills, patient responses to techniques, or ways to educate patients and families to perform
techniques. . . could add to knowledge needed for evidence-based practice.

DIF: Cognitive Level: Analysis REF: Page 73

2. Reasons to conduct an exact replication include which of the following? (Select all that
apply.)
a. A different sample is used in the replication, because subjects seldom elect to
undergo the same surgical procedure twice.
b. The same site is again used, in order to decrease variation.
c. Sample size was adequate, the design was strong, and measurements were robust.
d. Validation of the truthfulness of the original subjects’ responses is desired.
e. A similar population is used, in order to verify the findings.
ANS: B, C, D
Four different types of replication are important in generating sound scientific knowledge for
nursing: (1) exact, (2) approximate, (3) concurrent, and (4) systematic extension. An exact (or
identical) replication involves duplicating the initial researcher’s study to confirm the original
findings. All conditions of the original study must be maintained. Exact replications might be
thought of as ideal to confirm original study findings, but these are frequently not attainable.
In addition, one would not want to replicate the errors in an original study, such as small
sample size, weak design, or poor-quality measurement methods.

DIF: Cognitive Level: Analysis REF: Page 77

3. The American Association of Critical Care Nurses (AACN) funds various research projects
that focus on its research priorities. A master’s student wants to initiate research to study the
relative accuracy of new computer-assisted assessment device that painlessly measures blood
glucose values through a probe just distal to the insertion hub of a central line, in patients on
insulin drips with hourly Accu-Chek readings. Does this pertain to any of the organization’s
research priorities, listed here? (Select all that apply.)
a. Technology use to achieve patient assessment, management, or outcomes
b. Prevention and management of complications
c. Processes and systems that foster the optimal contribution of critical care nurses
d. Creation of a healing, humane environment
e. Development of processes and systems that foster the optimal contribution of
critical care nurses
ANS: A, D
The American Association of Critical Care Nurses (AACN) determined initial research
priorities for this specialty in the early 1980s and revised these priorities based on patients
needs and the changes in health care. The current AACN (2011) research priorities are
identified on this organization’s website as (1) effective and appropriate use of technology to
achieve optimal patient assessment, management, or outcomes; (2) creation of a healing,
humane environment; (3) processes and systems that foster the optimal contribution of critical
care nurses; (4) effective approaches to symptom management; and (5) prevention and
management of complications. This research study uses technology. Also, it saves the patient
from interruptions in sleep, as the nurse flushes a line and draws blood hourly, or performs
hourly Accu-Cheks, if the line will not draw.

DIF: Cognitive Level: Synthesis REF: Page 79

4. “The purpose of the research will be, most likely, to document how admirable charitable
efforts by The Children of the Land were terminated by the well-meaning Los Angeles Police
Force.” What is incorrect about this wording? (Select all that apply.)
a. A police organization cannot be named in a research purpose.
b. A purpose should specify methodology.
c. The terms admirable and well-meaning are both subjective.
d. The purpose must identify the goal of the study, not the “most likely” goal.
e. The purpose should be stated as “was” or “is” but not “will be.”
ANS: C, D, E
The purpose is generated from the problem, identifies the goal or goals of the study, and
directs the development of the study. In the research process, the purpose is usually stated
after the problem, because the problem identifies the gap in knowledge in a selected area and
the purpose clarifies the knowledge to be generated by the study. The research purpose must
be stated objectively, that is, in a way that does not reflect particular biases or values of the
researcher. Investigators who do not recognize their values might include their biases in the
research. This can lead them to generate the answers they want or believe to be true and might
add inaccurate information to a discipline’s body of knowledge.

DIF: Cognitive Level: Evaluation REF: Page 83

5. Children in publicly funded school breakfast programs often have learning delays. These are
not readily attributable to single causes. Research on learning delays has revealed that family
literacy, measured by parental reading level and comprehension scores, is the most powerful
predictor of delay in the primary grades. On the other hand, repeated exposure to eyes-on
reading, in the company of a trusted non-parent adult, has been shown to over-ride family
literacy as a predictor. No research, however, has studied institution of a reading-and-
breakfast program, delivered five days a week before school, intended to over-ride the
variable of family literacy. Given this problem statement, which of these purposes would be
appropriate for the study? (Select all that apply.)
a. The purpose of the study was to determine whether providing volunteer readers
during school breakfasts for all kindergarten and first-grade children would result
in fewer than anticipated learning delays.
b. The purpose of the study was to determine the lived experience of children with
learning delays, against the context of school and home, and to examine the
children’s peer relationships.
c. The purpose of the study was to determine whether a buddy system of one sixth-
grader, and one kindergartner or first-grader, who ate breakfast together and then
read together for 20 minutes, was effective in decreasing the anticipated number
learning delays.
d. The purpose of the study was to experimentally determine what causes learning
delays, by introducing various strategies already in place in community primary
schools and measuring their effect, using basic research.
e. The purpose of the study was to measure the effectiveness of using school
computers, allowing children to visually scan a story concurrently read by a school
teacher over the cafeteria microphone during school breakfast time, in decreasing
the incidence and severity of learning delays.
ANS: A, C, E
The problem statement identifies the specific gap in the knowledge needed for practice. Each
problem may generate many research purposes. The research purpose is a clear, concise
statement of the specific goal or aim of the study that is generated from the research problem.
The purpose usually indicates the type of study to be conducted and often includes the
variables, population, and setting for the study.

DIF: Cognitive Level: Synthesis REF: Page 73

6. In determining a study’s feasibility, which of the following statements are true, regarding the
time needed for study completion? (Select all that apply.)
a. Some data collection must be performed over an extended period of time, such as
measurements of the depth and extent of scar tissue over 18 months.
b. Sufficient subjects meeting the study criteria may be difficult to access, requiring
data collection that extends for months or even years.
c. The inflationary spiral makes all research funding inadequate.
d. Computerized records make data analysis much less time-consuming.
e. Obtaining Institutional Review Board (IRB. approval may be time-consuming,
especially if the research uses more than one hospital or agency.
ANS: A, B, E

One can approximate the time needed to complete a study by assessing the following factors:
(1) type and number of subjects needed, (2) number and complexity of the variables to be
studied, (3) methods for measuring the variables (are instruments available to measure the
variables, or must they be developed?), (4) methods for collecting data, and (5) the data
analysis process. Another factor that can increase the time needed for a study is obtaining
institutional review board (IRB) approval, especially if more than one clinical agency is used
for data collection in a study. Not all research funding is inadequate. Computerized records
may be easier or may be more difficult for data retrieval; however, data analysis is not
affected by the existence of computerized data.

DIF: Cognitive Level: Application REF: Page 84

7. How do seasoned nurse researchers, years away from clinical practice, select meaningful
research questions? (Select all that apply.)
a. They continue to conduct research in the same general areas in which they have
previously conducted research, with one project leading into the next.
b. They cannot do so, because they are not working bedside nurses.
c. They establish working relationships with clinical nurses, sharing authorship of
research publications.
d. They collaborate with novice researchers who are closer to the clinical world.
e. They use research methods that allow data collection through only observation.
ANS: A, C, D
Interactions with researchers and peers offer valuable opportunities for generating research
problems. Experienced researchers serve as mentors and help novice researchers to identify
research topics and formulate problems. This type of relationship could also be developed
between an expert researcher and a nurse clinician. Since nursing research is critical to be
designated as a Magnet facility, hospitals and healthcare systems employ nurse researchers for
the purpose of guiding studies conducted by staff nurses. Building an evidence-based practice
for nursing requires collaboration between nurse researchers and clinicians, as well as
collaboration with researchers from other health-related disciplines. During data collection,
study variables are measured through a variety of techniques, such as observation, interview,
questionnaires, scales, and physiological measurement methods.

DIF: Cognitive Level: Synthesis REF: Page 76

8. A master’s student does not know how to choose a research problem. She has been off work
for the past two years. What sources can she use in order to identify a researchable nursing
problem? Her work area used to be trauma nursing, but she does not wish to perform research
in this area. By using which of the following sources can she identify a nursing research
problem? (Select all that apply.)
a. Talking with nurse friends about questions that have arisen in their work areas
b. Talking with other master’s students about ideas for study
c. Reading professional research journals
d. Depending on her instructors to provide a research problem for her
e. National Institutes for Nursing Research priorities

ANS: A, B, C, E
The major sources for nursing research problems include nursing practice; researcher and peer
interactions; literature review; theories; and research priorities identified by individuals,
specialty groups, professional organizations, and funding agencies.

DIF: Cognitive Level: Application REF: Page 75

9. What is the relationship among the research problem, the research purpose, and the research
question? (Select all that apply.)
a. The purpose is but one of many purposes that can be generated from one particular
problem statement.
b. The problem statement is more focused and specific than is the purpose.
c. The research purpose and the research question should address the same facet of
the research problem.
d. The problem, purpose, and question are all focused upon a specific gap in the
knowledge base.
e. Research can be conducted without a research question, but not without a research
purpose.
ANS: A, C, D
A research problem is an area of concern where there is a gap in the knowledge based needed
for nursing practice. A research problem includes significance, background, and a problem
statement. The problem statement identifies the specific gap in the knowledge needed for
practice. The research purpose is a clear, concise statement of the specific goal or aim of the
study that is generated from the research problem, so the purpose is usually identified after the
research purpose is identified. Consequently, more than one purpose can emanate from a
given problem statement. Based on the research purpose, specific research objectives,
questions, or hypotheses are developed to direct the study.

DIF: Cognitive Level: Analysis REF: Page 73

10. A researcher gains support of the medical staff, the nursing staff, and the nurse manager of a
cardiothoracic ICU within a prestigious private hospital, aligned with a teaching institution. A
research proposal, concerning ambulation patterns after bypass surgery, is approved by the
Human Subjects Committee. Federal funding is obtained. Just before data collection is to
begin, the hospital is sold to a large university with a medical school, the nurse manager is
replaced with a manger from another hospital in the corporation, and there is a 30% staff
turnover. Choose the factors that are real concerns and could impact feasibility. (Select all that
apply.)
a. Most of the newly hired nurses are BSNs and newly graduated.
b. The new manager grudgingly allows the research to proceed but makes it clear that
she will not support subsequent research until the unit is more stable.
c. Fewer patients come to this hospital now for bypass surgery, going instead to its
sister hospital across town.
d. Two of the research assistants, who were already trained, take jobs elsewhere.
e. Staff nurses dislike the new manager and miss the old one.
ANS: B, C, D
The feasibility of a study is determined by examining the time and money commitment; the
researcher’s expertise; availability of subjects, facility, and equipment; cooperation of others;
and the study’s ethical considerations. A study might appear feasible but, without the
cooperation of others, it is not. However, most nursing studies involve human subjects and are
conducted in hospitals, clinics, schools, offices, or homes. Having the cooperation of people
in the research setting, the subjects, and the research assistants involved in data collection is
essential. People are frequently willing to cooperate with a study if they view the problem and
purpose as significant or if they are personally interested.

DIF: Cognitive Level: Analysis REF: Page 84

11. A master’s student decides to conduct a pilot study in order to help with which of the
following? (Select all that apply.)
a. Assess working nurses’ responses to having a researcher collect data in the middle
of their unit.
b. Re-evaluate the actual expenditure of the researcher’s time for each subject
consented and studied.
c. Allow potential subjects to sample research participation before they fully commit.
d. Apply for Human Subjects permission to perform the actual study.
e. Determine whether the research side has enough research subjects who are
interested in participating in the research.
ANS: A
A pilot study is commonly defined as a smaller version of a proposed study conducted to
refine the methodology. It is developed much like the proposed study, using similar subjects,
the same setting, the same treatment, and the same data collection and analysis techniques. As
the research problem and purpose increase in clarity and conciseness, the researcher has
greater direction in determining the feasibility of a study. The feasibility of a study is
determined by examining the time and money commitment; the researcher’s expertise;
availability of subjects, facility, and equipment; cooperation of others; and the study’s ethical
considerations. The purpose selected for investigation must be ethical, and Human Subjects
permission must be obtained before even a pilot study is conducted. A pilot study is not used
to allow potential subjects to sample participation before they commit: Human Subjects
permission must be obtained before study participation.

DIF: Cognitive Level: Analysis

MULTIPLE CHOICE

1. Which of the following would be landmark research?


a. The discovery that during pancreatitis, the pancreas is actually digested by its own
enzymes
b. A description of the importance of injecting immunizations in the proper location
of the deltoid
c. The third in a series of four papers describing patterns of emergency room use in
vacation communities
d. The first paper on the effect of using insulin for type I diabetes in humans
ANS: D
Seminal studies are the first studies that prompted the initiation of the field of research.
Landmark studies are the studies that led to an important development or a turning point in the
field of research.

DIF: Cognitive Level: Application REF: Page 100

2. A publication is printed every two months. Its volume number coincides with its year of
publication (2008 = 1; 2009 = 2; 2010 = 3; etc.). Its issue number coincides with the order of
publication, within a given year (Jan–Feb = 1; Mar–Apr = 2; etc). What kind of a publication
is this?
a. A monograph
b. A periodical
c. An e-book
d. A serial
ANS: D
Serials are published over time or may be in multiple volumes, but do not necessarily have a
predictable publication date. Periodicals are subsets of serials with predictable publication
dates, such as journals, which are published over time and are numbered sequentially for the
years published. This sequential numbering is seen in the year, volume, issue, and page
numbering of a journal. Monographs, such as books, hard-copy conference proceedings, or
pamphlets, are usually written once and may be updated with a new edition as needed.
Textbooks are monographs written to be used in formal education programs. Entire volumes
of books available in a digital or electronic format are called e-books.

DIF: Cognitive Level: Application REF: Page 100

3. The type of literature that describes concept analyses, models, and frameworks is which of the
following?
a. Empirical
b. Applicable
c. Able to be replicated
d. Theoretical
ANS: D
Theoretical literature consists of concept analyses, models, theories, and conceptual
frameworks that support a selected research problem and purpose. Empirical literature
comprises knowledge derived from research.

DIF: Cognitive Level: Application REF: Page 100

4. What primary sources might be available to someone writing a biography of Queen Elizabeth
I, who died in the 17th century?
a. An interview with one of her maids-in-waiting
b. A previous history written about her
c. An article about her in a 17th-century publication
d. A diary written by her
ANS: D
The published literature contains primary and secondary sources. A primary source is written
by the person who originated, or is responsible for generating, the ideas published. A research
publication published by the person or people who conducted the research is a primary source.
A theoretical book or paper written by the theorist who developed the theory or conceptual
content is a primary source. A secondary source summarizes or quotes content from primary
sources. Thus, authors of secondary sources paraphrase the works of researchers and theorists.
The problem with a secondary source is that the author has interpreted the works of someone
else, and this interpretation is influenced by that author’s perception and bias. Authors have
sometimes spread errors and misinterpretations by using secondary sources rather than
primary sources. You should use mostly primary sources to write literature reviews.
Secondary sources are used only if primary sources cannot be located or if a secondary source
contains creative ideas or a unique organization of information not found in a primary source.

DIF: Cognitive Level: Application REF: Page 101

5. What is the purpose of the minimal review of relevant studies that the grounded theory
researcher undertakes before writing the research proposal?
a. It compares the anticipated findings with the findings of similar research.
b. It directs the researcher in how to strategize data collection.
c. It helps the researcher to identify previous findings that will assist in interpretation
of the planned study.
d. It reveals the need for the planned research by identifying what others have done.
ANS: D
In qualitative research, the purpose and timing of the literature review vary based on the type
of study to be conducted. In development of a grounded theory study, a minimal review of
relevant studies provides the beginning point of the inquiry, but this review is only a means of
making the researcher aware of what studies have been conducted. This information, however,
is not used to direct the collection of data or interpretation of the findings in a grounded theory
study.

DIF: Cognitive Level: Application REF: Page 98

6. In terms of the literature review, how are quantitative research and ethnographic research
similar?
a. Both require the researcher to review the literature before beginning the study.
b. Both require the researcher to review the literature after completion of data
analysis.
c. Both require the researcher to utilize the literature as the primary data source.
d. Both consider the literature review extraneous, postponing it until after the study is
published.
ANS: A
In qualitative research, the purpose and timing of the literature review vary based on the type
of study to be conducted. The purposes for reviewing the literature for ethnographic studies
and for exploratory descriptive qualitative research are more similar to the literature review
for quantitative research. The researcher develops a general understanding of the concepts to
be examined related to the selected culture or topic. The literature review also provides a
background for conducting the study and interpreting the findings.

DIF: Cognitive Level: Analysis REF: Page 98

7. Considering phenomenologists’ belief that experience constitutes reality, how does their
approach to the literature review dovetail with that belief?
a. The literature is exactly as real as the other research data, and it is all analyzed and
valued equally during the data analysis portion of the study.
b. If the literature reports other phenomenologists’ findings, based on experience,
these can be considered alternative data sources.
c. The literature is a false interpretation of reality and cannot be considered, either
before or after data analysis is complete.
d. The literature review is usually postponed until after data analysis completion, so
that the understanding of the phenomenon will emanate solely from the data.
ANS: B
In qualitative research, the purpose and timing of the literature review depends on the type of
study to be conducted. Some phenomenologists believe the literature should not be reviewed
until after the data have been collected and analyzed so that the literature will not interfere
with the researcher’s ability to suspend what is known and approach the topic with openness.
DIF: Cognitive Level: Evaluation REF: Page 98

8. In historical research, what is the reason that the literature review begins so early and extends
so far into the process?
a. Historians must include exact dates in their histories. The literature must be
checked and double-checked, in order to verify the correctness of these dates.
b. Historical research demands one extra stage of data review, just prior to
publication, since new published data could bring the results under scrutiny.
c. Historians develop their ideas for research proposals from reading other histories.
As discrepancies arise, these provide the ideas for research.
d. The literature essentially comprises the bulk of the data set. From this, plus other
artifacts and interviews, if available, the historian writes the story.
ANS: D
In qualitative research, the purpose and timing of the literature review vary based on the type
of study to be conducted. In historical research, the initial review of the literature helps the
researcher define the study questions and make decisions about relevant sources. The data
collection is actually an intense review of published and unpublished documents that the
researcher has found.

DIF: Cognitive Level: Evaluation REF: Page 98

9. Why would the Boolean article OR be used if a researcher is conducting a digital literature
search of journals on the topic of prolonged adolescent grieving after parental loss?
a. It focuses the search on parental loss, the last search term.
b. It is useful when a researcher is undecided.
c. It narrows the search to articles containing all terms.
d. It allows the researcher to enter the search terms without excluding those whose
authors did not “keyword” all the words of the topic.
ANS: D
The Boolean operators are the three words AND, OR, and NOT. Often they must be
capitalized. The Boolean operators AND and NOT are used with the identified concepts. The
Boolean operator OR is most useful with synonymous terms or concepts. It is used to search
for the presence of any of a group of terms in the same search.

DIF: Cognitive Level: Application REF: Page 105

10. The original quotation in the fictional text by Atchison, Topeka, and Santa Fe (2009) is as
follows:
“Because of overemphasis on academic excellence, especially test grades, the high schools
studied had a disproportionate number of students who reported sleeplessness,
nervousness, nightmares, and guilt. These were attributed to various factors, the most
significant of which was a very strict principal, who voiced open disapproval of students
she felt were underachieving their enormous academic potential. Parents were very
accepting of this behavior, echoing it in their interactions with the students.”
What is the acceptable way to properly attribute this content in a literature review?
a. Parents were very accepting of the principal’s behavior, which included voiced
disapproval of underachieving students, echoing it in their interactions with their
sons and daughters.
b. Because of overemphasis on academic excellence, especially test grades, the high

You might also like